80
KAPLAN LSAT PREP LSAT RELEASED TEST XV EXPLAINED A Guide to the June, 1995 LSAT KAPLAN The answer to the test question.

15-PrepTest 15 Expls - dl.keywin.orgdl.keywin.org/3/4/340700517960b9bee3575f6766ff5765.pdf · • Sometimes the LSAT writers throw you a real curve when a Reading Comp ... LSAT Test

Embed Size (px)

Citation preview

Page 1: 15-PrepTest 15 Expls - dl.keywin.orgdl.keywin.org/3/4/340700517960b9bee3575f6766ff5765.pdf · • Sometimes the LSAT writers throw you a real curve when a Reading Comp ... LSAT Test

KAPLAN LSAT PREP

LSAT

RELEASED TEST XVEXPLAINED

A Guide to the June, 1995 LSAT

KAPLANThe answer to the test question.

Page 2: 15-PrepTest 15 Expls - dl.keywin.orgdl.keywin.org/3/4/340700517960b9bee3575f6766ff5765.pdf · • Sometimes the LSAT writers throw you a real curve when a Reading Comp ... LSAT Test

1995 Stanley H. Kaplan Educational Center Ltd

All rights reserved. No part of this book may be reproduced in any form, byphotostat, microfilm, xerography or any other means, or incorporated into anyinformation retrieval system, electronic or mechanical, without the written permissionof Stanley H. Kaplan Educational Center Ltd.

Page 3: 15-PrepTest 15 Expls - dl.keywin.orgdl.keywin.org/3/4/340700517960b9bee3575f6766ff5765.pdf · • Sometimes the LSAT writers throw you a real curve when a Reading Comp ... LSAT Test

© K A P L A N 1

SECTION I:

READING COMPREHENSION

Page 4: 15-PrepTest 15 Expls - dl.keywin.orgdl.keywin.org/3/4/340700517960b9bee3575f6766ff5765.pdf · • Sometimes the LSAT writers throw you a real curve when a Reading Comp ... LSAT Test

LSAT PREP ________________________________________________________________ LSAT Test XV Explained: Section I

2 © K A P L A N

PASSAGE 1—What Happened to the Dinosaurs?(Q. 1-7)

Topic and Scope: Dinosaur extinction; specifically, various different contemporarytheories for dinosaurs’ disappearance.

Purpose and Main Idea: The author spends most of the time explaining and touting thenewest of three theories (the “volcanic-eruption theory”) as to why the dinosaurs died out.Two earlier theories are described as well, but the author seems to have the mostconfidence in this new one.

Paragraph Structure: Each of the first three paragraphs is devoted to a thumbnaildescription of a theory: the “climatic theory” (¶1) that held sway until the 1980s; the“meteorite-impact theory” (¶2) popular in the ‘80s; and the “volcanic-eruption theory” (¶s3-4), which is supported by newly-discovered evidence in India. The introductorykeyword “Moreover” leads us to expect more information about the new theory, and that’sjust what we get: We learn that the evidence for the other two theories (the change in sealevel and the iridium deposits) supports, or at least is consistent with, the eruption theoryas well.

The Big Picture:

• Sometimes the LSAT writers throw you a real curve when a Reading Comp. sectionbegins—a really complex passage, or one that’s way beyond most everyone’s ken.Happily for the June, 1995 test takers, that wasn’t the case here. The passage’sstructure is as straightforward as can be, what with the theories and keyword“Moreover” clearly signaling the key shifts; and thanks to Jurassic Park, this ishardly a topic that most would find either unfamiliar or scary. On your LSAT, thetestmakers may not be so obliging. Just remember: At least one of those passageswill be of “low difficulty,” like this one, meaning that most people are expected to dowell on it. Find it, and attack it early in the section, even if it’s not printed first.

• Actually, the only examinees who screwed up were those who saw that the topic wasscience and, knee-jerk, hightailed it to another passage. A word to the wise: Evenscience passages can be manageable.

• Whenever the author describes several different points of view, take care to ascertainher views on them: Which, if any, does she especially favor? Here, it’s not too toughto detect the author’s enthusiasm for the volcano theory, but other passages may bemore challenging in this regard.

Page 5: 15-PrepTest 15 Expls - dl.keywin.orgdl.keywin.org/3/4/340700517960b9bee3575f6766ff5765.pdf · • Sometimes the LSAT writers throw you a real curve when a Reading Comp ... LSAT Test

LSAT PREP ________________________________________________________________ LSAT Test XV Explained: Section I

© K A P L A N 3

The Questions:

1. (C)This choice is a nice paraphrase of information in ¶2, which describes the nuts and bolts ofthe “meteorite-impact theory.”

(A) Au contraire. Lines 5-9 demonstrate that the “climactic theory”—a theory that precededthe “meteorite-impact theory”—could account for the extinction of ocean species at theend of the Cretaceous era.

(B) refers to the details of the “volcanic-eruption theory,” a new theory intended to take theplace of the older “meteorite-impact theory” of the 1980s.

(D) Au contraire aussi. ¶1 says that researchers initially thought that the drop in sea levelthought to be behind the mass extinction episode at the end of the Cretaceous era was theresult of “noncatastrophic geological processes.”

(E) mistakenly attributes an element of the “volcanic-eruption theory” to the “meteorite-impact theory.”

• When a passage discusses several different theories or scenarios, be sure that you’reclear about the details of each—the questions will certainly test to see that you are.

2. (A)The depths of the Earth’s mantle are described in two places, line 28 and line 40, and wehope you kept reading past the first reference because the right answer is yielded by thesecond. Lines 40-41 are choice (A) almost word-for-word.

(B) Only the element iridium is described here, and in fact that’s an element that is presentin the mantle but rare on the Earth’s surface. (B) gets it all cockeyed.

(C) According to lines 27-28, the mantle is rather unstable, and we get no information as towhether things are more stable closer to the surface.

(D) The only reference to CO2 comes in line 24, before the mantle is even mentioned, andall we’re told is that it’s released with lava. Yes, the lava eruption is triggered by mantleinstability, but that’s a long way from what (D) is saying.

(E) As with (C), we’re given no comparison between the mantle and the upper regions.And, anyhow, the mantle (when heated by the core) becomes less dense in places, not“uniformly” so.

• Use key nouns in question stems to help locate answers. Simply skimming for theword “mantle” helps you figure out the only places where the right answer can befound. However, be careful...

Page 6: 15-PrepTest 15 Expls - dl.keywin.orgdl.keywin.org/3/4/340700517960b9bee3575f6766ff5765.pdf · • Sometimes the LSAT writers throw you a real curve when a Reading Comp ... LSAT Test

LSAT PREP ________________________________________________________________ LSAT Test XV Explained: Section I

4 © K A P L A N

• ...because the first reference you find may not be the only or right one. Wheneveryou think you’ve located an answer, keep reading and searching a bit longer, to beon the safe side. Here, if you give up when you see line 28, you may become temptedby distortions in choices (B) and (E). But if you check further and light on line 40,your extra care pays off.

3. (D)Lines 39-43 say that the Earth’s mantle is rich in iridium, while its surface doesn’t havemuch of the stuff. Hence, we can infer that Cretaceous era lava (lava is just heated rockfrom the Earth’s interior) “was richer in iridium” than surface rock.

(A) is beyond the scope of the passage, which never compares Cretaceous era lava withmeteorites in terms of their respective carbon dioxide content.

(B) Au contraire. Lines 29-32 indicate that lava is no more dense than—and may well be lessdense than—the molten rock just above the Earth’s core.

(C) According to the “volcanic-eruption theory,” the Earth’s climate was changed bymassive volcanic eruptions, not by the release of iridium hexaflouride, which was simply abyproduct of those eruptions.

(E) The “volcanic-eruption theory” states that Cretaceous era lava distributed iridiummore evenly on the Earth’s surface than meteorite impacts could have; it doesn’t, however,claim that the lava was richer in iridium than meteorites.

• Don’t be taken in by choices like (A), (B), (C) and (E) here, which seem plausible on afirst read through, but which actually distort the content of the text.

4. (A)What could be more of a slam dunk? If you pre-phrased an answer after reading thepassage but before reading the choices, it’s possible that you came up with (A) almostword-for-word.

(B), (C) “Attacking”? “Inadequacies”? The author isn’t critical of any of the theories. She’sjust reporting on which theories have held sway when. And (B)’s implication that morethan one theory was popular before the 1980s is just inaccurate, as line 10 makes clear.

(D) implies that the topic and scope of the passage focus on general principles of scientificinquiry, and the relationship between evidence and theories. But no “general assertion” ismade; this passage is, as noted above, a straightforward examination of dinosaur extinctiontheories, based largely on chronology.

(E) is a mess. No “skepticism” is confirmed, and the “view held prior to the 1980s” is butone of the three theories explained, not the highlight of the passage.

Page 7: 15-PrepTest 15 Expls - dl.keywin.orgdl.keywin.org/3/4/340700517960b9bee3575f6766ff5765.pdf · • Sometimes the LSAT writers throw you a real curve when a Reading Comp ... LSAT Test

LSAT PREP ________________________________________________________________ LSAT Test XV Explained: Section I

© K A P L A N 5

• Many Reading Comp. questions are pretty easy, and many Reading Comp. wronganswers are pretty lame. Case in point. Even if you’re struggling with this section ingeneral, there will be questions that you can handle without much strain.

5. (D)This choice is an excellent paraphrase of the information contained in lines 42-47.

(A) is beyond the scope of the passage, which only discusses possible developments at theend of the Cretaceous era.

(B) According to the “volcanic-eruption theory,” the increase in atmospheric iridium andthe drop in sea level were both caused by volcanic activity; there was no direct connectionbetween the drop in sea level and the rise in atmospheric iridium levels.

(C) Again, the extinction of ocean species at the end of the Cretaceous era, according to thevolcanic theory, was due to volcanic activity, not to rising levels of iridium in theatmosphere.

(E) The “volcanic-eruption theory” never claims that iridium is released into theatmosphere through “normal geological processes.” This last phrase is associated with the“climactic theory” discussed in ¶1.

• If a question stem points you to a particular set of lines or a particular paragraph, goback and reread the relevant text before endorsing any of the choices. Otherwise youmight fall for a choice that’s wrong for a very subtle reason—a reason that you mighthave missed on an initial reading of the passage.

6. (B)Lines 54-57 say that, according to the “volcanic-eruption theory,” the gradual fall in sealevel was the result of the upward movement of diapirs toward the Earth’s surface.

(A) Lines 26-34 demonstrate that this theory makes assumptions about the temperature ofmolten rock just above the Earth’s core.

(C) is beyond the scope of the text, which doesn’t discuss episodes of mass extinctionbefore the one at the end of the Cretaceous era.

(D) While the “volcanic-eruption theory” explains the distribution of iridium on theEarth’s surface, it doesn’t try to explain either the relative scarcity of this element on theEarth’s surface or its relative abundance in meteorites.

(E) According to the “volcanic-eruption theory,” iridium should be distributed relativelyevenly in this layer of clay.

• When most of the questions in a set focus on one theory or issue or scenario, use theknowledge that you’ve acquired in answering the easier questions to help you withthe tougher ones.

Page 8: 15-PrepTest 15 Expls - dl.keywin.orgdl.keywin.org/3/4/340700517960b9bee3575f6766ff5765.pdf · • Sometimes the LSAT writers throw you a real curve when a Reading Comp ... LSAT Test

LSAT PREP ________________________________________________________________ LSAT Test XV Explained: Section I

6 © K A P L A N

7. (B)The “volcanic-eruption theory” claims that volcanic activity caused the climactic changesthat resulted in the mass extinction that occurred at the end of the Cretaceous era. If similarvolcanic activity had occurred at other times, but had not resulted in climactic changes,then we’d have to think twice about whether this theory is in fact on the right track.

(A) The “volcanic-eruption theory” claims that iridium hexaflouride comes from theEarth’s interior. Hence, the theory wouldn’t be called into question if meteorites werefound to have only minor quantities of the stuff. If anything, this evidence would tend tostrengthen the theory.

(C) The existence of other episodes of mass extinction would have no bearing on the“volcanic-eruption theory,” which (so far as we know, anyway) concerns only the massextinction at the end of the Cretaceous era.

(D) The “volcanic-eruption theory” doesn’t deny that meteorites hit the Earth at the end ofthe Cretaceous era; it simply claims that, however frequent these impacts may have been,they can’t account for the physical evidence associated with the mass extinction at the endof this era.

(E) If marine species are in fact more vulnerable to sudden changes in sea level than togradual changes, this fact would in no way undermine the notion that they could’vesuccumbed to a gradual change at the end of the Cretaceous era, as the “volcanic-eruptiontheory” suggests.

• In Strengthen/Weaken questions, be sure that you’re clear about whether you’rebeing asked to find the choice that strengthens or weakens the theory, scenario, etc.Why? Because you can be sure that if you’re asked, say, to find the choice thatstrengthens the theory or scenario, there’ll be wrong choices that weaken it, and vice-versa.

Page 9: 15-PrepTest 15 Expls - dl.keywin.orgdl.keywin.org/3/4/340700517960b9bee3575f6766ff5765.pdf · • Sometimes the LSAT writers throw you a real curve when a Reading Comp ... LSAT Test

LSAT PREP ________________________________________________________________ LSAT Test XV Explained: Section I

© K A P L A N 7

PASSAGE 2—Women and Folklore(Q. 8-15)

Topic and Scope: Folklore studies; specifically, a recent shift in interest to womenfolklorists.

Purpose and Main Idea: The author sets out to provide evidence as to the increasedinterest in women folklorists. While the passage begins with a description of another shift—to “the folk” (performers) and away from just “the lore” (stories)—notice that the authornever gets back to that until ¶4. The bulk of his interest is women performers.

Paragraph Structure: ¶1 lays out the topic, scope, and shifts described above, and ¶2provides specifics as to the shift to interest in women: a specific element of what’s been truein folklore “until recently” (compare line 2 to line 13), and a specific example of how thingsare different.

¶3 first takes us back a century—to show that this interest in women folklorists isn’t uniqueto the present day—and then highlights the content, pros, and cons of two contemporarystudies of two women folklorists. ¶4’s focus is defined by lines 52-53—brief speculation asto the potential “result[s] of this line of study”—both the increased emphasis on womenand the interest in “the folk” rather than just “the lore.”

The Big Picture:

• Early on you should listen keenly for keywords that hint at overall structure. Here,for instance, “It has become...a truism...until recently” clearly tells you that, in theauthor’s mind, that truism no longer holds. Figure out the truism and you have theauthor’s p.o.v. in the bag!

• You must listen for, recognize, and interpret keywords throughout your reading, ofcourse, not just at the beginning.

• Many students reported some initial confusion as to what “the lore was often morestudied than the folk” meant. LSAT authors will often toss in a figure of speech likethis—simply to see whether it throws you! Of course, the author immediately adds“That is,” (line 3)—indicating that the phrase is about to be redefined. (And, ofcourse, this is all something of a curve, because the author spends more time on therole of women than on the folk vs. lore stuff anyhow.)

• Always remember: If a difficult phrase or concept is important enough, the authorwill define it more clearly for you before too long. (And if it’s not important, whocares what it means?)

Page 10: 15-PrepTest 15 Expls - dl.keywin.orgdl.keywin.org/3/4/340700517960b9bee3575f6766ff5765.pdf · • Sometimes the LSAT writers throw you a real curve when a Reading Comp ... LSAT Test

LSAT PREP ________________________________________________________________ LSAT Test XV Explained: Section I

8 © K A P L A N

The Questions:

8. (E)This choice nicely captures the author’s topic, scope, and purpose.

(A) According to lines 7-9, folk performers, as opposed to just their works, began to getattention from the early 1970s. The increased emphasis on women, the text implies, is aneven more recent phenomenon. Besides, the precise date when scholars first began to giveattention to the contributions of women folklorists is a mere detail.

(B) nicely sums up the gist of ¶4, but certainly doesn’t qualify as the passage’s main point.

(C) Lines 38-48 indicate that recent works about women folklorists don’t focus primarilyon the “the problems of repertoire analysis.”

(D) The text claims that folklore studies have shifted from an almost exclusive emphasis onfolklore itself to an interest in the people who transmit it. (D) makes a quite different claim.

• In global questions, you’ve got to look for the choice that comes to grips with theentire passage. Steer clear of choices that focus on individual details or ¶s.

9. (E)¶2’s purpose, as noted above, is to provide examples of the new trend toward focusing onwomen in folklore studies, and the author asserts that The Dynamics of Folklore is a “telling”example—“perhaps more telling,” even, than several recent studies in which women arecentral.

(A) What’s “too soon to tell” is the extent of the shift (line 18-20), but the author has no suchreservations as to whether women are coming to the forefront.

(B) Weigle & Farrer are looking backwards to traditional folklore collection, and TheDynamics is an example of the new trend, so the latter couldn’t possibly be there to “refute”the former. In any case, lines 18-21 act as a transitional sentence clearly separating the twodetails.

(C) Wrong ¶. Repertoire analysis doesn’t show up until ¶3.

(D) may be highly tempting to less than careful readers, but ¶2 is solely about the newinterest in women folklorists. It sidesteps the “performer vs. material” distinction that’sraised in ¶s 1 and 4 and highlighted in (D).

• As you read, your favorite keywords should be Emphasis Keywords (like “Perhapsmore telling”), because they will always help you to differentiate what truly mattersto the author from the factual and the tangential.

• As often as not, a “purpose of a reference” question like this one is a “purpose of theparagraph” question. Remember what the paragraph in question is there for, andyou can usually get a quick point.

Page 11: 15-PrepTest 15 Expls - dl.keywin.orgdl.keywin.org/3/4/340700517960b9bee3575f6766ff5765.pdf · • Sometimes the LSAT writers throw you a real curve when a Reading Comp ... LSAT Test

LSAT PREP ________________________________________________________________ LSAT Test XV Explained: Section I

© K A P L A N 9

10. (C)The folklorists mentioned in the last ¶ are concerned with studying folklore in its largercultural “context,” and in understanding the different ends to which it is put by men andwomen. The book in (C) reflects these two interests.

(A) overlooks the male/female distinction made by current folklore researchers.

(B) gets the male/female distinction right, but misses the cultural emphasis of currentscholarship.

(D) reflects the historical emphasis on the substance of folklore itself to the exclusion ofthose who transmit it.

(E) distorts a detail from the wrong ¶—¶1.

• In choices that ask you to go outside of the passage itself, look for the choice that isanalogous to or parallels the relevant information in the passage.

11. (B)This choice captures the essence of lines 7-10.

(A) Folklore research has recently been concentrating on the differences, not the similarities,in the ways that men and women use folklore.

(C) This choice distorts a detail from the wrong ¶—¶3.

(D) Au contraire. Folklore research has increasingly sought to place folklore in its propercultural context.

(E) Since the early 1970s, there has been an increasing scholarly interest in individuals whotransmit folklore.

• Note how you could have used the date (the 1970s) in the question stem to zero in onlines 7-10, where the answer to this question is to be found. In general, use whateverinformation the question stem gives you to focus your search for the correct answer.

Page 12: 15-PrepTest 15 Expls - dl.keywin.orgdl.keywin.org/3/4/340700517960b9bee3575f6766ff5765.pdf · • Sometimes the LSAT writers throw you a real curve when a Reading Comp ... LSAT Test

LSAT PREP ________________________________________________________________ LSAT Test XV Explained: Section I

10 © K A P L A N

12. (A)Early folklorists, remember, concentrated on the folklore itself, and not on the people whotransmitted the stuff. Why? Because these folklorists felt that the performers were notcreative people who contributed to the material itself.

(B) is beyond the scope of the passage, which discusses scholarship about performers offolklore, not the views of the performers themselves.

(C) is also beyond the scope of the passage, which, again, focuses on scholarship about folkperformers. There’s no real discussion about folklore itself, let alone whether it underwentchange from generation to generation.

(D) distorts the gist of lines 13-18, which state that early folklorists tended to downplay thecontributions of women performers. That’s quite different from saying that early folkloriststhought that women didn’t have much of a role in transmitting folklore.

(E) The male/female distinction regarding the meaning of a piece of folklore was not onemade by early folklorists. This is a distinction made by contemporary scholars.

• The correct answer to any inference question will always stick very close to the spiritof the text. If you have to work very hard to endorse a choice, it’s probably wrong.

13. (A)Before the early 1970s, folklorists studied folklore but paid no attention to the people whowere involved in transmitting it. Likewise, the anthropologist in (A) studies “implements”(i.e., tools) but pays no attention to the people who used those implements.

(B)-(E) None of these choices reflects the basic pattern of pre-1970s scholarship on folklore:an emphasis on a “cultural product” combined with a lack of interest in those who actually“use” the product.

• Note how this question probes the exact same issue as the previous question. It’squite common on LSAT Reading Comp. for two or three questions in a set to dealwith the same issue; so be on the lookout for connections among questions. Useyour answers to the easier questions to help you solve the tougher ones.

Page 13: 15-PrepTest 15 Expls - dl.keywin.orgdl.keywin.org/3/4/340700517960b9bee3575f6766ff5765.pdf · • Sometimes the LSAT writers throw you a real curve when a Reading Comp ... LSAT Test

LSAT PREP ________________________________________________________________ LSAT Test XV Explained: Section I

© K A P L A N 11

14. (D)According to the last ¶ of the text, current folklore scholarship is concerned with analyzingthe links between specific pieces of folklore and the larger society from which it emerged.Hence, “context” here refers to the “environment” and “circumstances” surrounding apiece of folklore.

(A), (C), and (E) are all beyond the scope of the passage, which doesn’t deal either with theinternal relationships among different parts of a piece of folklore or with the connectionbetween folklore and physical locale.

(B) The text doesn’t delve into the issue of successive interpretations of the same piece offolklore by different scholars.

• “Vocabulary-in-context” questions are uncommon on the LSAT, but you may seeone on test day. If you do, read the lines around the word or phrase in question toget a sense of how it’s used in the text.

15. (B)Abraham’s book is “notable” (line 43) but “unfortunately” (line 46) lacks a key element, andthat amounts to less than 100% endorsement.

(A) The “approval” cannot be “wholehearted” given lines 46-48.

(C), (D), and (E) all ignore the reference to Abraham’s book being “notable.” (Moreover,they’re all so close in meaning that it’s almost impossible to choose among them.)

• When two or more choices are functionally identical, none of them can be correct,because the right answer must be categorically correct and the other fourcategorically wrong.

Page 14: 15-PrepTest 15 Expls - dl.keywin.orgdl.keywin.org/3/4/340700517960b9bee3575f6766ff5765.pdf · • Sometimes the LSAT writers throw you a real curve when a Reading Comp ... LSAT Test

LSAT PREP ________________________________________________________________ LSAT Test XV Explained: Section I

12 © K A P L A N

PASSAGE 3—Pocock and Political Discourse(Q. 16-21)

Topic and Scope: Pocock’s approach to political discourse; specifically, his use oflinguistic analysis to interpret the political discourse of the past.

Purpose and Main Idea: The author’s purpose is to describe and critique Pocock’smethodology. His specific main idea is that Pocock’s methodology is a valuable tool forunderstanding past political discourse, even if it doesn’t fully explain each and everyhistorical document.

Paragraph Structure: ¶1 describes Pocock’s basic methodology and compares it(favorably) to traditional methods of interpreting political texts of the past. ¶2 describesPocock’s work: how he has applied his methodology to investigate “civic humanism” ineighteenth-century England.

¶3 continues the discussion of Pocock’s work, noting that his analysis of political discoursein eighteenth-century America doesn’t ring as true as his analysis of English politicaldiscourse. Nevertheless, the author ends the passage by saying that, although Pocock’swork isn’t entirely on the mark, his methodology is certainly on the right track.

The Big Picture:

• The passage begins unpromisingly. Take the opening sentence, for instance: Manyexaminees’ heads were swimming as early as line 7. Note, too, the sleight-of-hand interms of topic and scope: Pocock disappears from the passage until line 22, yet turnsout to be the central figure. This kind of thing is very rare in LSAT passages. Andthe passage stays about as difficult as it begins.

• The answer, of course, is to get a lot of mileage out of the other three passages—which do turn out to be more manageable—and try to budget your time so that youend up having more than the average 8 mins. or so when you finally take thispassage on.

Page 15: 15-PrepTest 15 Expls - dl.keywin.orgdl.keywin.org/3/4/340700517960b9bee3575f6766ff5765.pdf · • Sometimes the LSAT writers throw you a real curve when a Reading Comp ... LSAT Test

LSAT PREP ________________________________________________________________ LSAT Test XV Explained: Section I

© K A P L A N 13

The Questions:

16. (C)This choice neatly captures the author’s topic, scope, and purpose; and it clearly echoesthe last sentence of the passage in its critical approval of Pocock’s work.

(A), (B) “Civic humanism” (A) and “eighteenth-century political texts” (B) are certainlyprominent aspects of this passage. But the main focus of this text is on Pocock’smethodology and its application to eighteenth-century political texts. Yet neither of thesechoices even mentions Pocock.

(D) concentrates on a detail that emerges only in ¶3.

(E) plays on a detail in ¶1.

• The answer to many questions in a Reading Comp. question set often ties intoinformation at the end of the passage. So, a word of advice: Don’t just skim over thelast sentences of the passage on the assumption that they don’t contain importantinformation.

17. (C)The association of English whigs and “a vocabulary of economic progress” is explicitlymade in lines 32-34, but this passage is so dense that you are perhaps to be forgiven if youmissed it.

(A) Tempting if your outside associations of Jefferson label him as a populist or“progressive,” but we’re specifically told that, to Pocock, Jefferson echoed Tory vocabulary(lines 42-45). The author disagrees, of course, but the focus of the question is Pocock’s view.

(B) To make this choice work you have to go to way too much trouble. Once you hearPocock label Jefferson a closet Tory (lines 42-45), you have to assume that Jefferson’sopponents, the Federalists, were therefore closet Whigs, and then assume that theFederalists must therefore have adopted the Whig political vocabulary mentioned aparagraph ago. Yikes! Far less risky to grab the explicit right answer in lines 32-34.

(D), (E) Au contraire. The Tories are cited as the antithesis of the Whigs, and the rurallandowners are identified with the former. Both groups’ vocabulary, inferably, is that “ofpublic spirit and self-sufficiency” (lines 29-30), not “economic progress.”

• This is a rare LSAT question in that we are asked to do little more than locate adetail. Why is it rare? Because it’s usually so easy to find an explicit reference thatthey don’t waste a question on it. Why do we get one here? Because this passage isunusually impenetrable, so quickly finding anything specific in it is evidence ofskill. Nothing happens by accident in the creation of an LSAT.

Page 16: 15-PrepTest 15 Expls - dl.keywin.orgdl.keywin.org/3/4/340700517960b9bee3575f6766ff5765.pdf · • Sometimes the LSAT writers throw you a real curve when a Reading Comp ... LSAT Test

LSAT PREP ________________________________________________________________ LSAT Test XV Explained: Section I

14 © K A P L A N

18. (D)To the author, “Pocock’s ideas have proved fertile” (lines 39-40), and thus it is fitting “toapplaud the historian [that’s Pocock, in case you have fallen asleep by the end] who” (lines59-60) while not 100% successful, has made political vocabularies a vital topic. Theauthor’s attitude, then, is much more positive than negative.

(A) 1/2 right, 1/2 wrong. “Fruitful” is positive enough as a characterization of one ofPocock’s key assumptions, but “cant” (line 39) is Namier speaking of political language inthe 1700s, and is way too negative to ascribe to the author as a judgment on Pocock.

(B) “Sharp” is positive, but as used here it describes a “contrast’ (line 16), not acharacteristic of Pocock. And “elitist” at line 46 is Jefferson’s characterization of EnglishTories.

(C) 1/2 right, 1/2 wrong, but in reverse order. “Controversial” is applied to some ofPocock’s theories, but the “naiveté” is that of the theorists who preceded Pocock.

(E) “Importance” sounds okay, but in line 55 it’s used in connection with differenteighteenth-century political vocabularies, not Pocock. “Simply” doesn’t have eitherpositive or negative overtones.

• If you kept in mind the author’s generally positive attitude toward Pocock’s work,you should have zeroed in on choice (D) rather quickly.

19. (B)This choice nicely paraphrases the author’s criticism (lines 17-21) of the method of politicalanalysis used in the 1950s.

(A) The keyword “while” (line 13) indicates that the author contrasts the assumptions ofthe 1950s with the way literary historians derive the meaning of a political text.

(C) is beyond the scope of the passage, which never suggests that political texts can be“read in...different ways” depending on one’s philosophic bent.

(D) This is an assumption made by Pocock, not analysts of the 1950s.

(E) The author is critical of the 1950s methodology for assuming that historical knowledgewasn’t relevant to the interpretation of documents.

• Whenever you’re given a line number in the question stem, the answer to thequestion will be in the lines around that number. Don’t endorse any choice thatstrays too far from the relevant portion of text.

Page 17: 15-PrepTest 15 Expls - dl.keywin.orgdl.keywin.org/3/4/340700517960b9bee3575f6766ff5765.pdf · • Sometimes the LSAT writers throw you a real curve when a Reading Comp ... LSAT Test

LSAT PREP ________________________________________________________________ LSAT Test XV Explained: Section I

© K A P L A N 15

20. (D)In lines 41-54, the author argues that Pocock erred in applying the same “linguisticdichotomy” to both England and America. What works for England, the author asserts,doesn’t necessarily work for America.

(A) is beyond the scope of the passage. Pocock concerns himself with political discourseonly.

(B) It’s the author, not Pocock, who “denigrates” the role of analytic philosophers inanalyzing political texts.

(C), (E) The author wholeheartedly endorses Pocock’s interpretation of eighteenth-centurypolitical discourse in England.

• This is an excellent example of why it’s important to keep track of the gist of eachparagraph of the passage. If you remembered that the author critiques Pocock onlyin the last paragraph, you could have gone straight there and quickly discovered that(D) is correct.

21. (A)¶1 describes Pocock’s basic method of studying political discourse; ¶s 2 and 3 describe theapplication of this method to the cases of eighteenth-century England and America; and ¶3evaluates the merits of Pocock’s method in light of its application to these cases. (A)reflects this sequence.

(B) The author has reservations about Pocock’s work, and he states them after apresentation of the evidence.

(C) What hypothesis? This passage describes and evaluates a scholarly mode of inquiry.

(D) The author does evaluate Pocock’s work and does suggest a future direction forresearch, but this choice says nothing about all of the description of Pocock’s work.

(E) What comparisons and contrasts? What categories of evaluation? What framework?

• If you’ve got a good grasp of the structure and purpose of the passage, questionswith abstract-sounding answer choices won’t be difficult to decipher.

Page 18: 15-PrepTest 15 Expls - dl.keywin.orgdl.keywin.org/3/4/340700517960b9bee3575f6766ff5765.pdf · • Sometimes the LSAT writers throw you a real curve when a Reading Comp ... LSAT Test

LSAT PREP ________________________________________________________________ LSAT Test XV Explained: Section I

16 © K A P L A N

PASSAGE 4—Black Economic Progress(Q. 22-27)

Topic and Scope: Black economic progress; specifically, the effects civil rights laws andeducational opportunities have had on Black economic progress.

Purpose and Main Idea: The author’s purpose is to counter an argument regarding Blackeconomic progress. Her specific main idea is that civil rights laws, not increasededucational opportunities, account for recent Black economic progress.

Paragraph Structure: ¶s 1 and 2 are descriptive in nature. ¶1 outlines the relevant civilrights laws, while ¶2 explains the “continuous change” hypothesis, which argues that Blackeconomic progress should be attributed to improved educational opportunities ratherthan civil rights laws.

¶s 3 and 4, to the contrary, are argumentative in nature. In ¶3, the author cites severalreasons to explain why educational opportunities can’t account for Black economicprogress. In ¶4, she claims that the economic progress made by Blacks since the mid-1960sis directly attributable to the civil rights legislation of the mid-1960s.

The Big Picture:

• Not a promising place to begin work on the Reading Comp. section. Why? Theauthor’s voice doesn’t enter the picture until ¶3. The first couple of ¶s bombard youwith a lot of detail without giving you any clear signals about where the passage isheaded. In general, it’s best to work first on passages where topic, scope, andauthorial purpose are clear early on. These passages lend themselves to scoringquick and easy points.

• When a passage contains a lot of details, it’s easy to get bogged down in them. Don’tlet this happen to you on test day. Remember, you’re only going to get questionedabout one or two details. As you read through the passage, move past detailsquickly, noting where they appear, so that you can look them up easily should youhave to.

The Questions:

22. (E)Title VII prohibits all employers “from making employment decisions on the basis of race.”Executive Order 11,246, on the other hand, concerns only government contractors. In otherwords, Title VII “governs hiring practices in a wider variety of workplaces.”

(A), (B) It’s Executive Order 11,246 that calls for monitoring employers to ensure minorityrepresentation (A) and deals with government contractors (B).

(C) and (D) are beyond the scope of the passage, which never brings up the issues of wagediscrimination (C) or minority representation in government (D).

Page 19: 15-PrepTest 15 Expls - dl.keywin.orgdl.keywin.org/3/4/340700517960b9bee3575f6766ff5765.pdf · • Sometimes the LSAT writers throw you a real curve when a Reading Comp ... LSAT Test

LSAT PREP ________________________________________________________________ LSAT Test XV Explained: Section I

© K A P L A N 17

• If you reread ¶1, this question yields a quick and easy point. If you try to answer ona vague recollection of the text, you can quickly lose that point. If you’re unsureabout a detail, reread, reread, reread!

23. (B)Lines 30-32 state that, in the mid-1940s, Blacks were catching up to Whites in amount ofschooling, but were not yet equal. (B) makes precisely the same point in a different way.

(A), (C), and (D) are all beyond the scope of the passage. There’s no mention of Whiteschool expenditures (A), Black or White school curriculums (B), or the general quality ofWhite schools (D).

(E) 1/2 right, 1/2 wrong. Lines 34-35 say that teachers at Black schools did experience wageincreases in the mid-1940s, but we aren’t told whether wages increased at a greater or lesserrate than wages for teachers at White schools.

• In inference questions, it’s common to find choices that address issues that thepassage doesn’t discuss. Make sure that the choice you endorse is in fact dealt within the text.

24. (C)The author’s specific main idea, as we’ve already mentioned, is that Black economicprogress should be attributed to civil rights legislation, not to better educationalopportunities. In making this argument, she rebuts the “continuous change” hypothesis,which asserts just the opposite.

(A) The author contends that the “continuous change” hypothesis is incorrect, not that it’sincomplete.

(B), (D) The author does indeed discuss both the “impact of education” (B) and “Blackeconomic progress before and after the 1960s” (D), but these are mere details intended tosupport her larger argument.

(E) The author provides her own perspective about the factors behind Black economicprogress, not the “current view.”

• A simple “verb scan” eliminates choices (B) and (D), because this passage isargumentative, not descriptive. A “verb scan” can often be very helpful in narrowingdown the choices in global questions.

Page 20: 15-PrepTest 15 Expls - dl.keywin.orgdl.keywin.org/3/4/340700517960b9bee3575f6766ff5765.pdf · • Sometimes the LSAT writers throw you a real curve when a Reading Comp ... LSAT Test

LSAT PREP ________________________________________________________________ LSAT Test XV Explained: Section I

18 © K A P L A N

25. (C)This choice neatly summarizes the essence of lines 12-16.

(A), (D), (E) All we’re told about continuity theorists is that they don’t believe thatlegislation accounts for recent Black economic progress; we aren’t told anything regardingtheir general attitude about legislation’s effects on discrimination.

(B) Continuity theorists claim that Blacks have made progress in education, but they don’tascribe this progress to the law.

• Correct choices will always stick close to the spirit of the text. If you have to gothrough a lot of mental gymnastics to justify a choice, look for another choice.

26. (A)This concession is sandwiched between information intended to bolster the claim that civilrights laws have contributed to Black economic progress. Hence, its meant to strengthenthat claim by explaining away a possible objection to it.

(B) The only cause of Black economic progress that the author cites is civil rightslegislation.

(C), (D) The author concedes nothing to the continuity theorists (C). Nor does she alter herargument in light of their hypothesis (D).

(E) This is scope of the entire passage, not just the lines that come after 60.

• When a question asks about the why of a detail, read the lines around the detail itselfto get a sense of the context in which it appears. This is the key to understandingwhy the author put the detail in the text.

27. (D)The “continuous change” hypothesis claims that Black economic progress is unrelated togovernment actions. Similarly, (D) features a scenario in which progress is made withoutgovernment assistance.

(A), (B), (C), (E) In each of these scenarios, a problem is solved through governmentintervention.

• If one choice differs from the rest in a fundamental way, it’s a good bet that it’s thecorrect answer.

Page 21: 15-PrepTest 15 Expls - dl.keywin.orgdl.keywin.org/3/4/340700517960b9bee3575f6766ff5765.pdf · • Sometimes the LSAT writers throw you a real curve when a Reading Comp ... LSAT Test

© K A P L A N 19

SECTION II:

LOGICAL REASONING

Page 22: 15-PrepTest 15 Expls - dl.keywin.orgdl.keywin.org/3/4/340700517960b9bee3575f6766ff5765.pdf · • Sometimes the LSAT writers throw you a real curve when a Reading Comp ... LSAT Test

LSAT PREP _______________________________________________________________ LSAT Test XV Explained: Section II

20 © K A P L A N

1. (A)By providing a rationale for prohibiting an otherwise legal activity—namely, harm towhich others must be exposed—(A) justifies Walter’s recommendation for the airlines.Smoking is that otherwise legal activity, and the nonsmokers on a plane are the “others”who can’t avoid harm.

(B) “Only if” is the tipoff here. Walter’s purpose isn’t to describe a precondition for a banon smoking, but to recommend a ban in one specific situation. Even if (B) matched up wellto every element of Walter’s argument—which it doesn’t, because the reference to “mostsituations” has no connection to what Walter says—(B) still wouldn’t justify his proposal,but would simply establish a condition necessary for a smoking ban.

(C) The “legal activity” would have to be smoking, but how would one “modify” it? Byusing a cigarette holder? C’mon.

(D) is a justification for keeping smokers out of planes, not for keeping planes free ofsmoke.

(E) is a justification for making an activity, presumably smoking, “legal in all situations.”This one is way off.

• A “principle” question is generally nothing more than a matter of matching up thetopic, scope, and terms of the argument to the choices.

• In such questions, be sure to translate carefully the abstract elements of the answerchoices to the specific elements of the stimulus.

2. (D)“. . . your challenge is ineffectual, since you are simply jealous. . . .” Those nine words areall you need to answer this “logical flaw” question, and having read them you should havejumped to the choices and grabbed (D). One is never permitted to rebut an opponent bycriticizing his or her personality, character, or motives. Doing so is called an ad hominemattack, but LSAC doesn’t expect you to know the Latin term; (D) is how the testmakerstypically word it.

(A) The chemist restates no claim, and the use of the Evidence Keyword “since” establishesthat he does offer evidence. It’s not effective evidence, but it’s evidence nonetheless.

(B) is only tempting if you decide that since the physicist believes that “measurements andcalculations are inaccurate,” the chemist must believe that they are accurate. But as we’venoted, the chemist disagrees with the physicist on personal grounds, not on the grounds ofaccuracy.

(C) Nope, in this context “solve” has only one meaning: to locate the answer to a problem.

(E) To “rest on a contradiction” means to base your argument on two premises that cannotboth be true. The chemist does contradict, or take issue with, the physicist, but there are noconflicting premises within the chemist’s argument.

Page 23: 15-PrepTest 15 Expls - dl.keywin.orgdl.keywin.org/3/4/340700517960b9bee3575f6766ff5765.pdf · • Sometimes the LSAT writers throw you a real curve when a Reading Comp ... LSAT Test

LSAT PREP _______________________________________________________________ LSAT Test XV Explained: Section II

© K A P L A N 21

• An accusation of personal attack is often made in LSAT answer choices when thequestion is “What’s the flaw?” Select it only if the speaker does, in fact, slam his/heropponent. If there’s no personal attack, the speaker is committing some other flaw.

3. (E)The argument proposes a cause-and-effect relationship: Because ulcer patients possessedthis bacteria strain, and someone accidentally ingested the strain and got his first ulcer,therefore the strain probably brings on ulcers. Hard to prove, but easy to rebut: If one wereto find test subjects who possess the bacteria strain but no ulcer, that would certainly tossthe theory out the window, wouldn’t it? Now, (E) found no such test subjects out of a largesample: Many people without ulcers were examined and no traces of the bacteria strainwere found. Hence, (E) supports the reasoning by short-circuiting a major possibleexception.

Think of it in formal logic terms. The stimulus essentially argues that “If you have thebacteria strain, then you’ll get an ulcer,” the contrapositive of which is “If you don’t havean ulcer, then you won’t have the bacteria strain.” And there they are in (E), 2,000 peoplewho fit that definition—no ulcer, and no bacteria either.

(A) A secondary disease is outside the scope of an investigation of whether a particularcause results in a particular effect.

(B) The stimulus doesn’t argue that the bacteria causes only ulcers, so the presence orabsence of other ailments is irrelevant.

(C) Even if we accept that one can learn about human diseases from studying otheranimals—a notion that comes from without the stimulus, not from within—the absence ofulcers in (C) would weaken the case for the causal agent, not strengthen it.

(D) appeals to an expert’s authority. Phooey. Experts can be wrong.

• To strengthen an argument doesn’t mean to prove it. It means to strengthen theconnection between evidence and conclusion. One way to do so, as in Question 3here, is to counter a possible objection to that connection.

• LSAT wrong answers often appeal to authority. Never choose answers like (D)—unless, of course, the stimulus argument is appealing to authority and you’re askedfor the logical flaw.

4. (D)The stimulus conclusion—to which we seek a parallel—is hypothetical: Should one of thesetest subjects own a pet, he or she would have a lower average blood pressure. The evidence?Lowered blood pressure apparently caused by petting animals. (D) has it right. (D) usesthe relaxing caused by a short boat ride in the same way that the stimulus uses the lowerblood pressure caused by animal petting: as evidence for the hypothesis that ownership ofan object (a boat and a pet, respectively) would cause a general improvement of the trait(relaxing and lower b.p., respectively).

Page 24: 15-PrepTest 15 Expls - dl.keywin.orgdl.keywin.org/3/4/340700517960b9bee3575f6766ff5765.pdf · • Sometimes the LSAT writers throw you a real curve when a Reading Comp ... LSAT Test

LSAT PREP _______________________________________________________________ LSAT Test XV Explained: Section II

22 © K A P L A N

(A) is only superficially similar to the stimulus. Its evidence lacks the sense of one actionhaving a particular effect on one limited group of people, and its conclusion fails to allegethat ownership of an object would have a greater effect.

(B),(C) Each errs by focusing on a policy recommendation—retaining a car and dumpingcertain plants, respectively. The stimulus never recommends anything; it predicts whatwould happen (lower blood pressure) if the test subjects owned a pet.

(E), like (A), may be tempting because of its hypothetical conclusion. But in the stimulus,petting an animal and owning a pet are two different things—which is in fact the root of theflawed logic: Since they are different things, one cannot assume that the salutary effectcaused by petting an animal would be repeated, let alone intensified, by pet ownership.But (E)’s one coat vs. two coats lacks that difference. Two coats of paint probably wouldmake for a whiter fence.

• Your #1 tactic in Parallel Reasoning questions should always be to explore the natureof the evidence and conclusion. What kind of conclusion is drawn, and what kind ofevidence used?

5. (A)This discrepancy (or “paradox”) hinges on the definition of “best bill collector.” How canhe be the best if his collection rate is the worst? Maybe you were able to predict the answerand maybe you weren’t, but we hope you recognized it as soon as you saw it: The reasonhis collection rate is so bad is that he’s assigned to the hardest-core cases—as would befitthe best employee in the firm.

(B) is akin to (D) back in Question 3. What Young’s co-workers think of him is as irrelevantto this paradox as the expertise of the stomach researcher is to the cause of ulcers.

(C) deepens the paradox, by suggesting that his crummy rate of collections is in fact hisnorm.

(D) is laughably irrelevant—though if you chose (D) you’re probably not laughing. We’renot told whether Young’s credit dept. job involved collections, and if it did, whetherYoung was good or bad at it. Not that any of that would make any difference to hisperformance at this job.

(E) Length of tenure is irrelevant too, having nothing to do with Young’s collection ratebelying his status as the best collector.

• A paradox usually emerges when the arguer makes a faulty assumption. Identify thatassumption and you usually will have resolved the paradox. And speaking ofassumptions. . . .

Page 25: 15-PrepTest 15 Expls - dl.keywin.orgdl.keywin.org/3/4/340700517960b9bee3575f6766ff5765.pdf · • Sometimes the LSAT writers throw you a real curve when a Reading Comp ... LSAT Test

LSAT PREP _______________________________________________________________ LSAT Test XV Explained: Section II

© K A P L A N 23

6. (B)The author sees a bit of a paradox here: Ancient primates, such as the one whose jawbonewas found in Namibia, lived in dense forests, yet today Namibia is dry and treeless. Toresolve this paradox, the author concludes that Namibia’s landscape must have changedover the centuries, but that assumes that that ape did, in fact, live in or near Namibia—choice (B). If it didn’t—if it wandered into Namibia far from its forested homeland andthereupon expired, or if perhaps the jawbone was blown by the wind or carried bysomeone to that spot—then Namibia’s terrain need not have changed at all. Since (B), iffalse, would weaken the argument, it is a necessary assumption.

(A) Modern apes are outside the scope of this argument. It’s modern Namibian terrain andancient apes that are at issue.

(C) Certainly the author believes that at least one ape—the one whose jawbone wasfound—lived in Namibia between 10 and 15 million years ago. But apes prior to 15 millionyears ago are outside the scope.

(D) Why apes lived in dense forests may or may not have anything to do with their diet.And (D), like (A), errs by mentioning modern apes.

(E) provides an explanation as to how the change in Namibian terrain might have takenplace, but it’s far from the only possible one. In any event the author is arguing aboutwhether the change occurred, not how.

• Always use the Kaplan “Denial Test” to confirm whether you have indeed chosen anecessary assumption. Note that the Denial Test is described above, beginning withthe words “If it didn’t. . . .”

7. (B)Is job-related stress the #1 workplace problem? Our author concludes no, it’s not, becausemost workers complain about boredom, not job stress. But if stress and boredom aresomehow related, then there’s no contradiction, and that’s essentially what we get in (B). (B)implies that workers who explicitly complain about boredom are implicitlydemonstrating signs of stress; hence the two go hand in hand; hence stress is probably the#1 problem.

(A),(D),(E) Non-complaining workers are outside the scope here, since the issue is “What’sthe most serious workplace problem?,” and the source of the evidence is those who aremaking their complaints known. Workers who are relatively happy, for whatever reason,aren’t part of this debate.

(C) doesn’t discuss complainers, either—just responders. And the “recentness” ofresponses has nothing to do with the argument.

Page 26: 15-PrepTest 15 Expls - dl.keywin.orgdl.keywin.org/3/4/340700517960b9bee3575f6766ff5765.pdf · • Sometimes the LSAT writers throw you a real curve when a Reading Comp ... LSAT Test

LSAT PREP _______________________________________________________________ LSAT Test XV Explained: Section II

24 © K A P L A N

• Keep an eye out for an argument whose flaw is that the author assumes anopposition between X and Y without sufficient evidence; by showing that X and Ycan coexist, you will have weakened the argument. (Here, the author assumes thatworkers cannot be complaining about boredom and stress simultaneously. But theycan!)

8. (A)Keywords are the key to structure, and structure is the key to figuring out this author’sconclusion quickly.

A yes/no question is posed: Should the government stop trying to figure out how muchtoxin is O.K. in our food? Three potent Keywords govern what’s left, starting with “Onlyif,” which as always signals a necessary condition—necessary, that is, for a Yes answer to thequestion. “However” signals a contrast—i.e. a statement that that necessary condition hasnot been fulfilled. And “furthermore” is a Continuation Keyword, which means “more ofthe same.” Feel free to read more deeply into the stimulus if you like, but the sheerstructure leads to only one conclusion: A condition necessary for abandoning thegovernment’s efforts has not been met, hence the government should not abandon itsefforts—choice (A). (Note that (A) substitutes “should continue” for “should notabandon”—a common testmaker tactic.)

(B) is the “Only if” clause—the condition necessary, in the author’s view, for thegovernment to abandon its efforts.

(C) is a very close paraphrase of the “However” clause—a piece of evidence suggestingthat the necessary condition has not been met.

(D) is an equally close paraphrase of the “Furthermore” clause—more evidence.

(E) cannot be the author’s conclusion, because the issue is not “Are the government’sdetection methods refined enough?”; no evidence either way is provided. (E) is amisreading of the very last clause of the stimulus. Inferably the author would probablyapprove of the government’s raising “the threshold of detection,” but only because of herconclusion that detection needs to continue. . . which brings us back to (A).

• The techniques outlined in the explanation above are well worth your studying andadopting for virtually every LR stimulus: (1) Put the key issue or conclusion intoyour own words; (2) notice the Keywords that govern the structure; and (3) thinkthrough what each Keyword portends. That’s critical reading! You’ve heard aboutthese techniques throughout your Kaplan course; perhaps seeing them worked outhere will prove helpful. And notice how similar this one is to Question 11, below.

Page 27: 15-PrepTest 15 Expls - dl.keywin.orgdl.keywin.org/3/4/340700517960b9bee3575f6766ff5765.pdf · • Sometimes the LSAT writers throw you a real curve when a Reading Comp ... LSAT Test

LSAT PREP _______________________________________________________________ LSAT Test XV Explained: Section II

© K A P L A N 25

9. (E)The stimulus for this inference question is more complex than most, but all four wronganswers are arguably more obvious than most, so perhaps it balances out.

Over a quarter-century of labor-saving technologies, we’re told, workers have been able toproduce goods (“output”) a lot faster, with the potential result that a worker could put infewer hours and get more leisure time. However, there has not been correspondence: Theaverage output per hour has grown twice as fast as the average worker’s leisure time. (E)follows from that last sentence: If the average worker is producing goods a lot faster butisn’t working a lot fewer hours, then he must be producing more per week than he did aquarter century ago, prior to the introduction of all those technologies.

Don’t see it? Check out the lameness of the other four choices:

(A) Nothing is ever said about workers’ spending habits or abilities.

(B) Job creation is never mentioned.

(C) The percentage of all persons who work is never even alluded to.

(D) “Anticipated” by whom? Nothing is mentioned about what was in people’s minds 25years ago. (And even if “anticipated” is read as a paraphrase of “potentially,” all we’re toldis that the number of hours worked was potentially smaller and the number of leisurehours potentially greater. Nothing about output expectations whatsoever.)

Sherlock Holmes had it right: When all other possibilities have been eliminated, what’s left,however implausible, must be the truth.

• Never assume that an inference must combine all, or even most, of the statements inthe stimulus. Quite often—as here—the correct answer is just a rewrite orconsequence of one complex sentence.

10. (C)Yet another paradox. A bunch of ancient Asian communities stopped hunting-and-gathering and started cultivating their food. Their diet and health went to blazes comparedto the old days, and yet they never went back to hunting-and-gathering. How come? Pre-phrasing an answer might be tough, especially if you’re unfamiliar with the archaeologicalterminology, but we hope that your search of the choices yielded (C) reasonably readily.If, as (C) suggests, there were just too many people to make hunting-and-gathering viable,why indeed would those ancient Asians go back to it, notwithstanding their dietaryproblems?

(A), if anything, would provide a reason for the Asians to resume hunting-and-gathering—to grab all that wild flora and fauna.

(B)’s contrast of stored vs. fresh foods has nothing to do with the argument at hand, and inany case describes something that hunter-gatherers and agriculturalists had in common,which doesn’t do the job.

Page 28: 15-PrepTest 15 Expls - dl.keywin.orgdl.keywin.org/3/4/340700517960b9bee3575f6766ff5765.pdf · • Sometimes the LSAT writers throw you a real curve when a Reading Comp ... LSAT Test

LSAT PREP _______________________________________________________________ LSAT Test XV Explained: Section II

26 © K A P L A N

(D) rather comically implies that the Asians failed to return to hunting-and-gathering inorder to keep up with the Joneses elsewhere in the world. Huh?

(E) seems to imply that ancient Asians chose agriculture over hunting-and-gatheringbecause the former burned more calories and hence would make them buff up. But thatmakes no sense: Remember that the agricultural peoples had a terrible diet and bad healthcompared with the old hunter/gatherers. Concern for their well being would seem tomandate returning to the old ways—which they failed to do. So (E), if anything, deepensthe paradox.

• Speculating about what the right answer might look like, contain, or be is a goodhabit, but don’t take it to extremes. If nothing comes to you right away, don’t standstill. Move briskly to the choices and assess them in whatever order strikes you aseasiest.

11. (E)Just like Q. 8, this one begins with a question that the author is apparently going to attemptto answer. There are only three possibilities allowed by the author to begin an article withthe phrase “in a surprise development”: the development was a surprise to the journalistalone, the development was a surprise to some other person, and the development was asurprise to people in general. In each case, the author says that the set phrase isinappropriate. The argument’s conclusion, therefore, is (E): In no case is it a goodjournalistic practice to begin a paragraph with the phrase “in a surprise development.”

(A) and (B) The journalist never describes circumstances in which it is appropriate to usethe phrase “in a surprise development.”

(C) Although the author does describe three distinct sorts of circumstances in which thephrase is used, that’s not his conclusion; he makes that distinction in order to help himarrive at his conclusion, that in no circumstances is it appropriate to use the phrase.

(D) This is way off; the author never discusses what’s permissible in summing up a story; heonly considers the phrase “in a surprise development” as an introduction, and even therehe finds it wanting.

• A wrong choice can be wrong for more than one reason; (D) for instance, besidesintroducing the idea of a summation, doesn’t deal with the specific phrase “in asurprise development,” which is what the author is interested in, but instead speaksof the practice of making the general point that a development is a surprise, which isa very different thing. This is a great example of a scope shift.

• The more logically a stimulus is organized, the easier it is to answer the question.This is a dream stimulus: it begins with the question the argument is supposed toanswer, which immediately tells you what the conclusion is going to be about. Itproceeds to an orderly examination of all the possible cases that might bear on thequestion, complete with Keywords and Keyword phrases (“the one remainingpossibility”).

Page 29: 15-PrepTest 15 Expls - dl.keywin.orgdl.keywin.org/3/4/340700517960b9bee3575f6766ff5765.pdf · • Sometimes the LSAT writers throw you a real curve when a Reading Comp ... LSAT Test

LSAT PREP _______________________________________________________________ LSAT Test XV Explained: Section II

© K A P L A N 27

12. (C)Despite the technical jargon, this stimulus is actually made up of a bunch of formal logicstatements. Take the third sentence: If X (yellow to black), then either Y (polypyrroles formon zeolites) or Z (polypyrroles form in zeolites). The last sentence tells us that X occurred(yellow to black), but Y didn’t. From this we can deduce that Z must have occurred:polypyrroles formed from pyrroles inside the zeolite. The second sentence tells us thatwhen polypyrroles form inside the zeolites, they form in delicate chains. Therefore, we caninfer (C): some of the pyrroles in which the zeolite was submerged formed intopolypyrrole chains.

(A) The stimulus says that the zeolite was “free of any pyrrole” before it was submerged,so (A) is impossible.

(B) Since the polypyrroles must have formed in and not on the zeolite, they must havebeen formed as chains, not lumps.

(D) There’s no reason why some pyrrole couldn’t attach itself to the zeolite; all we know isthat no polypyrroles formed from pyrroles on the zeolite.

(E) Since the polypyrroles formed inside the zeolite, we know some pyrroles reached thezeolite’s inner channels; we have absolutely no reason to infer that only a “little” did so.

• Don’t try to swallow this whole stimulus at one gulp. Read it quickly first to get thestructure. Once you understand that the stimulus gives us a general descriptionfollowed by a specific case, the whole picture becomes much easier to understand --you see how the general rules about zeolites and pyrroles relate to the specific case,and the individual facts fall into a pattern.

• You don’t need to know anything about polypyrroles and zeolites; it doesn’t evenmatter if there are no such things. All you need to do is understand the relationshipsbetween the objects, which in this case take the form of familiar formal logicconditions.

13. (E)The author says that the pedigree standards applied to working dogs ignore those genetictraits that allow the dogs to serve the purpose for which they were originally bred; as aresult, those practical traits may be lost. She decides that the standards set by pedigreeorganizations for working dogs should include the ability to do the work for which theywere intended. We’re looking for a principle that supports this conclusion. (E) says thatorganizations that set standards for activities or “products” (and presumably pedigreedogs fit this designation) should ensure that they can serve their original purposes; i.e.according to (E), the author is right and pedigree organizations should ensure that“working” dogs are still able to work.

(A) talks about the sort of standards organizations should not set, but it doesn’t justify theauthor’s conclusion about what standards pedigree organizations should set.

Page 30: 15-PrepTest 15 Expls - dl.keywin.orgdl.keywin.org/3/4/340700517960b9bee3575f6766ff5765.pdf · • Sometimes the LSAT writers throw you a real curve when a Reading Comp ... LSAT Test

LSAT PREP _______________________________________________________________ LSAT Test XV Explained: Section II

28 © K A P L A N

(B) The author isn’t in favor of enforcing those standards currently in effect, but favorssetting new standards.

(C) argues that organizations should see that their standards are met; the author is attackingthe prior question of what sort of standards should be set in the first place.

(D) The author doesn’t favor standards requiring working ability because today’spedigree dogs will actually be required to work; she argues in favor of the standardsbecause they reflect the original purpose of the breeds.

• Before you look at the choices, see if you can follow the author’s reasoningyourself—how does she move from what is to what should be? Here you might say:Why does the author think standards should require working ability? What does theauthor say about working ability? Well, it’s the reason for which working dogs wereoriginally developed, and it’s in danger of being lost. So the author thinks it’simportant to keep the original purpose . . . therefore, (E) stands out as the rightanswer.

14. (B)The author is attempting to establish the conclusion that standards set by pedigreeorganizations for working dogs should require working ability. The phrase “certain traitslike herding ability risk being lost among pedigreed dogs” is intended to support thisconclusion. The phrase itself is supported, since it occurs at the end of a chain of reasoning:Pedigree organizations don’t specify work traits among their standards; breeders onlymaintain traits specified by pedigree organizations; traits not maintained by breeders riskbeing lost; therefore work traits like herding ability risk being lost. As (B) says, the phrase isa “subsidiary” conclusion; that is, it’s a conclusion drawn on the way to the mainconclusion.

(A) On the contrary—as we’ve seen, it’s supported by several pieces of evidence.

(C) Far from acknowledging a possible objection to the author’s proposal, the phrase statesthe main reason for the author’s proposal.

(D) is dead wrong. The argument accepts the claim that traits like herding ability risk beinglost; the author’s proposal is intended to avert this very real danger.

(E) No earlier claim depends on the phrase; rather, the phrase is supported by earlier claims,and in turn gives support to the conclusion.

• Keep an eye on Keywords; the “since” in the second-to-last sentence tells you thatthe phrase in question appears as a conclusion, which points to (B) and rules out (A).

• In order to understand where any given phrase fits in an argument, you have tounderstand how the argument as a whole works. Hopefully, you figured this out enroute to answering Q. 13, in which case Q. 14 shouldn’t take very long.

Page 31: 15-PrepTest 15 Expls - dl.keywin.orgdl.keywin.org/3/4/340700517960b9bee3575f6766ff5765.pdf · • Sometimes the LSAT writers throw you a real curve when a Reading Comp ... LSAT Test

LSAT PREP _______________________________________________________________ LSAT Test XV Explained: Section II

© K A P L A N 29

15. (C)You may not have been able to pre-phrase this “strongly-supported conclusion,” buthopefully choice (C) made sense to you upon evaluation. The arthritis medication worksby inhibiting the functioning of the hormone that causes pain and swelling. However, thehormone doesn’t only cause pain and swelling in cases of rheumatoid arthritis; it’s alsonormally activated in response to injury and infection. It therefore seems reasonable tosuppose that the medication would inhibit the normal action of the hormone in response toan injury. This in turn supports (C)’s contention that a person taking the medication mightsustain a joint injury and, because of a lack of the pain and swelling that normallyaccompany such an injury, be unaware of it.

(A) There’s no evidence that the medication would repair cell damage that has alreadyoccurred; quite the contrary, since the only action ascribed to the medication is that ofinhibiting the functioning of the hormone.

(B) No mention of harmful side effects is made in the argument, so it’s impossible to knowwhether or not any side effects would be outweighed by benefits.

(D) We have no idea whether the hormone inhibited by the medication has anything to dowith diabetes or lupus, so there’s no reason to conclude that the medication would be ofany use in combating these diseases.

(E) On the contrary, we’d expect the medication to have some effect on any joint diseaseinvolving the production of the hormone that causes pain and swelling. We don’t knowthat there are any other such diseases besides rheumatoid arthritis, but we certainly can’tconclude that there aren’t.

• Medicine is a common topic on the Logical Reasoning section and “side effect”choices, like (B), are very common on medical questions. Unless the stimulusspecifically mentions side effects, you can’t assume anything about them. (By thesame token, the author isn’t entitled to assume anything about side effects either;sometimes on a Weaken the Argument question a choice will bring up the fact thatthe author has overlooked the possibility of side effects.)

• Pay attention to the formality, or lack thereof, of the question stem. They’re justlooking for a choice that is “strongly supported” here; the correct answer need not beas strict as a logical inference that absolutely must be true. Notice how the correctanswer is nicely qualified by the word “could.”

Page 32: 15-PrepTest 15 Expls - dl.keywin.orgdl.keywin.org/3/4/340700517960b9bee3575f6766ff5765.pdf · • Sometimes the LSAT writers throw you a real curve when a Reading Comp ... LSAT Test

LSAT PREP _______________________________________________________________ LSAT Test XV Explained: Section II

30 © K A P L A N

16. (E)The author has concluded that for the amaryllis plant, going dormant is a positive thing,and that therefore an amaryllis plant’s owner should actually induce dormancy in his plant.But the evidence doesn’t support this; it seems much more likely that going dormant issimply the amaryllis plant’s defense against the drought that occurs in its natural habitat. Ifso, then without a drought there’s probably no need for a period of dormancy. As (E)recognizes, the author assumes that there’s more to dormancy than a defensive reaction,that the dormancy confers some other positive benefit on amaryllis plants.

(A), (B) The author only speaks of amaryllis plants, and so need not assume anything aboutother plants or how other plants compare to amaryllis plants.

(C) The author never specified when water should be withheld from amaryllis plants keptas houseplants, only that it should be done at some time each year to induce dormancy.The lack of specificity in the argument regarding the time frame for withholding waterkills this one.

(D) Although the author believes dormancy is good for amaryllis plants, he needn’tassume that the only thing that could go wrong with an amaryllis plant is too short a periodof dormancy.

• An assumption is something that’s necessary to establish the conclusion; therefore,it can’t go beyond the scope of that conclusion. You can quickly reject (A) and (B)because they broaden the scope of the argument from amaryllis plants, the author’sonly interest, to other kinds of plants, about which the author says nothing.

17. (D)The author accepts the theory that yawning is most powerfully triggered by seeingsomeone else yawn because 1) a lot of people believe the theory today and 2) according to“historians of popular culture” people all over the world have commonly believed thetheory. That’s not a very convincing argument; we’d like to have something more thanwidespread belief to fall back on. That’s the flaw (D) points out; the argument relies onopinion, whereas the question under consideration—“what triggers yawning?”—is ascientific, or factual question, the answer to which needs to be supported by substantivefactual evidence.

(A) The author doesn’t indulge in circular argumentation—his evidence, albeit weak, iscertainly different from his conclusion.

(B) The author never defines “historians of popular culture,” but there’s no reason tosuppose that popular beliefs about yawning are outside their area of expertise.

(C) The author refers to common popular belief; he makes no reference to particular cases,either typical or atypical.

(E) The author concludes that seeing other people yawn is the “most irresistible” cause ofyawning; he never claims, or assumes, that it’s the only cause of yawning.

Page 33: 15-PrepTest 15 Expls - dl.keywin.orgdl.keywin.org/3/4/340700517960b9bee3575f6766ff5765.pdf · • Sometimes the LSAT writers throw you a real curve when a Reading Comp ... LSAT Test

LSAT PREP _______________________________________________________________ LSAT Test XV Explained: Section II

© K A P L A N 31

• Whenever you see a reference to “popular” or “commonly-held” belief on the LSAT,it’s a good bet that the belief is being set up to take a fall. A good argument is notsupposed to rely on common opinion, but rather on logic.

18. (B)The stimulus in this Parallel Reasoning question is so formal in nature that it can be boileddown to algebra:

All A are B (All gourmet cooks enjoy variety).

No B is C (No one who enjoys variety prefers bland food).

Therefore, no C is A (or if you prefer, All C are not A, same difference: The conclusion isthat gourmet cooks and preferrers of bland food have nothing in common).

In correct choice (B), Huang Collection paintings are A, abstract paintings are B, andpaintings included in next week’s auction are C, and if you plug those terms into thealgebra above, you’ll see that (B) functions identically. Here’s how the wrong choices playout:

(A) All A are B. All B are C. Therefore, all A are C. This is the classic Aristotelian deductivesyllogism, by the way. The sheer absence of a negative or “not” term means that you couldhave rejected (A) right away.

(C) All A are B. No A is C. Therefore, no C is B. Note the second premise’s subtle shift awayfrom the original.

(D) All A are B. No C is B. (So far so good, actually. Remember that No B is C is the sameas No C is B. They’re reversible.) Therefore, no B is C. (Oops; close but no cigar.)

(E) All A are B. . . . and here algebra falls short, because the remainder of (E) brings incomplex terms like prices in general, adequate reflection of price, true value, and nextweek’s auction. Because (E) is not as strictly written as the stimulus, it, like (A), can bequickly rejected.

• Parallel Reasoning questions that lend themselves to an algebraic approach don’tcome along all that often. Most are like Q. 4 in this section and Qs. 13 and 22 in theother Logical Reasoning section, written less formally and hence not amenable to analgebraic treatment. When algebra can work, it’s a great time saver.

• A reminder: In formal logic, “No” statements (e.g. No B are C) and “Some”statements (e.g. Some X are Y) are reversible—it doesn’t matter in which order youmention the terms. Come up with a few real-life examples of each and you’ll see thatthat’s so. But we cannot reverse All statements or If/Then statements and expect themto remain true. The LSAT tests that key distinction often.

Page 34: 15-PrepTest 15 Expls - dl.keywin.orgdl.keywin.org/3/4/340700517960b9bee3575f6766ff5765.pdf · • Sometimes the LSAT writers throw you a real curve when a Reading Comp ... LSAT Test

LSAT PREP _______________________________________________________________ LSAT Test XV Explained: Section II

32 © K A P L A N

19. (A)Because no one witnessed the meeting between the minister and the opposition partyleader except for the minister’s aide, the author concludes that it must have been theminister’s aide who provided the information that brought the minister down, rather than apolitical enemy of the minister. Does this make sense? After all, the opposition leader wasat the meeting too; mightn’t he or she have given the information to the newspapers? As (A)points out, the evidence provided by the author, makes a competing conclusion—that theopposition leader spilled the beans—just as likely as the author’s conclusion.

(B) The author never states that the chain of events was inevitable; that is, he never assumesthat, once the information from the secret meeting was released, it was inevitable that theminister be brought down.

(C) The author never discusses any “different occasion”; both evidence and conclusionconcern this particular finance minister and this secret meeting.

(D) The evidence is entirely relevant to the point at issue, which is “who was responsiblefor bringing the finance minister down?”

(E) The evidence is that the effect (the minister’s downfall) was impossible without theaction (giving the information to the newspaper). This means that the latter was necessaryfor the former; the author never treats the evidence of the action as though it was sufficientto bring about the effect.

• The descriptions of the flaw are all very abstract and general. Because it can be timeconsuming in such cases to compare each choice to the stimulus argument, it’s wiseto read the argument carefully enough to have a good idea of the flaw beforeattacking the choices. Pre-phrasing something as simple as “Hey! What about theopposition party leader?” can be an enormous help.

20. (A)Evans discounts those critics who dismiss him and his poetry, because he says theythemselves are not true poets and only a true poet can function as a critic of poetry. Whydoes he say that they’re not true poets? He’s read their poetry and doesn’t think much ofit—it doesn’t “convey genuine poetic creativity,” as true poetry should. He’s relying on hisown judgment of poetry; therefore, he’s assumes that he’s a fit judge of poetry and poeticcreativity, and according to his own criteria, he must also be assuming that he is a truepoet. As (A) points out, he’s assuming exactly what he set out to prove, that the critics arewrong and he’s a true poet.

(B) Evans never assumes or implies that everyone is either a poet or a critic. All she says isthat if you’re not one, you can’t be the other.

(C) Evans never implies that a poet’s standing can be judged independently of his or herpoetry; in fact, the criteria given for a true poet is one whose works conveys genuine poeticcreativity.

Page 35: 15-PrepTest 15 Expls - dl.keywin.orgdl.keywin.org/3/4/340700517960b9bee3575f6766ff5765.pdf · • Sometimes the LSAT writers throw you a real curve when a Reading Comp ... LSAT Test

LSAT PREP _______________________________________________________________ LSAT Test XV Explained: Section II

© K A P L A N 33

(D) Far from making an absolute distinction between critics and poets, the author says thatonly true poets can function as critics of poetry.

(E) On the contrary, true poets would be in a position to criticize their own work.

• It’s always important to understand what sort of evidence the author uses, especiallyon questions that ask you to critique the author’s logic. Our evidence here is ageneral principle (about who is entitled to judge poetry) and the author’s opinionabout how the principle applies to his critics. You should always be suspicious of anopinion as evidence. Ask yourself: How well grounded is the opinion? Howqualified is the person (or persons) holding the opinion?

21. (E)Ouch!—one of the nastiest question stems we’ve come across in a long time. If there wasever a question worth blowing off temporarily strictly based on its stem, this is it. Be that asit may, let’s take the stem apart to see what they’re after here.

The correct choice will be able to serve as the premise of an argument against the claim,but it must be an argument that relies on the principle. So we’re not looking for somethingthat by itself weakens the claim, but rather the basis for an argument that, if based on theprinciple, would weaken the claim. Tricky, eh? According to the principle, if one claimsthat Country X lowered its taxes to serve the advantage of foreign companies, one mustshow how the foreign companies’ interests actually helped bring about the change; that is,one must show that the reason for the tariffs’ being lowered was the usefulness of thatcourse of action to foreign companies. Since we’re supposed to counter the claim thatCountry X’s tariffs were lowered in order to help foreign companies, we should thereforelook for a choice that weakens the connection between foreign companies and the changein policy. The best choice is (E); if there’s no evidence that foreign companies themselvesplayed a role in inducing the change then, according to the principle, it becomes muchmore difficult to explain the change by the fact that it served their interests.

(A) The principle says nothing about the interests of any second group, like consumers,being served, so (A) doesn’t apply to the principle.

(B) introduces new economic considerations like importers’ profits; the principle hadnothing to do with such things, only with who was responsible for inducing the change.

(C) discusses the short-term effects of the change on Country X; we’re only interested inwhether the change occurred in response to foreign interests.

(D) is better than (C) because at least it tells us about foreign companies, but it doesn’t tellus what we wanted to know—whether they helped bring about the change.

• This question was very likely tough and time consuming; you may decide to mark it,skip it, and come back to it later.

Page 36: 15-PrepTest 15 Expls - dl.keywin.orgdl.keywin.org/3/4/340700517960b9bee3575f6766ff5765.pdf · • Sometimes the LSAT writers throw you a real curve when a Reading Comp ... LSAT Test

LSAT PREP _______________________________________________________________ LSAT Test XV Explained: Section II

34 © K A P L A N

• If you decided to go ahead with it, read the question thoroughly and understand itbefore you go on to the choices. If you took the time to understand what type ofchoice you were looking for, you could reject the wrong answers quickly bychecking them against the principle.

22. (E)The scientist discovers that in the tropics, migratory fish generally follow a migrationpattern that’s exactly the opposite of the pattern followed by fish in temperate zones: theymature in fresh water and spawn in the ocean. Does this disprove the scientist’s theory thatfood availability determines migration patterns? That’s impossible to know, unless weknow something about the food availability in the tropics—is there more or less foodavailable in the ocean? Unless we know the answer to (E), we don’t know whether the fishin the tropics are acting in accordance with the scientist’s theory or in violation of thattheory.

(A) completely ignores the issue of food availability which is the whole basis of the theory;the scientist said nothing about temperature, so (A) is irrelevant.

(B) introduces an irrelevant distinction; the scientist was interested in comparing the totalamount of nourishment available in the different places. The point is that fish need a lot ofnourishment when they mature and little when they spawn— it doesn’t matter what type offood they eat to get that nourishment.

(C) Because the scientist is talking about general migration patterns, it doesn’t matter ifthere are a few exceptions to the rule. Thus, if there were a species of fish with populationsin both areas, it wouldn’t make any difference to the theory whether the fish exhibited thesame or different migration patterns in the two different zones; what matters is what fish“generally” do.

(D) is pretty plainly irrelevant. The total number of species has nothing to do with the roleof food in determining migration patterns.

• Since you want to know whether the new information disproves the hypothesis,focus on the terms in the information and the hypothesis. The hypothesis tiesmigration patterns to food availability; the new information only discusses migrationpatterns. The correct choice must therefore establish whether there’s any linkbetween the new information and food availability.

• In a question like this, if you can’t decide whether one of the choices is relevant, tryanswering the question it poses in both the affirmative and the negative. Does itmake any obvious difference to the hypothesis if you answer the question “yes”rather than “no”? If it doesn’t, move on.

Page 37: 15-PrepTest 15 Expls - dl.keywin.orgdl.keywin.org/3/4/340700517960b9bee3575f6766ff5765.pdf · • Sometimes the LSAT writers throw you a real curve when a Reading Comp ... LSAT Test

LSAT PREP _______________________________________________________________ LSAT Test XV Explained: Section II

© K A P L A N 35

23. (B)According to the author, computers can only solve problems by following some set ofmechanically applicable rules, and some problems just can’t be solved that way; it followstherefore that no computer can solve that type of problem. But the author takes this onestep further; she says that no computer will be able to do everything that some human mindcan do. She must be assuming that some human mind can do what she’s just shown nocomputer can do—that is, she assumes that some human mind (or “at least one” humanmind, as (B) says) can solve one of those problems that can’t be solved by following amechanically applicable set of rules.

(A) The author needs to establish that some non-mechanical problems are solvable byhumans; it wouldn’t help to assume that there’s some non-mechanical problem that’sUNsolvable by humans.

(C), (D) and (E) all make the same mistake. Since computers also have the ability to solveproblems by following mechanically applicable rules, human ability to solve suchproblems doesn’t show that humans can do something that computers can’t. It wouldn’thelp the author to assume that one such problem can be solved by every human (C), thatevery such problem (of a certain sub-type) is solvable by almost every human (D), or thatevery such problem is solvable by some human (E). We’re only interested in human abilityto solve problems that can’t be solved by following sets of mechanical rules.

• Remember your logical terms: “Some” means “at least one”; “no” means “not any,”“not one.”

• Be on the look out for inappropriate use of “some,” “every,” “many,” etc. in thechoices. When the stimulus speaks of “some human minds” you can be almostpositive that any choice that speaks of “every human mind” or even “almost everyhuman mind” is wrong. “Some” strictly means “at least one”; you can’t expand it. Bythe same token, choices that apply such terms where they don’t occur in the stimulus(“more than one” set of rules) should also be rejected.

Page 38: 15-PrepTest 15 Expls - dl.keywin.orgdl.keywin.org/3/4/340700517960b9bee3575f6766ff5765.pdf · • Sometimes the LSAT writers throw you a real curve when a Reading Comp ... LSAT Test

LSAT PREP _______________________________________________________________ LSAT Test XV Explained: Section II

36 © K A P L A N

24. (A)The author is trying to demonstrate that the typical response given to the survey isambiguous. He does this by taking that typical response and applying it to the case of atheoretical representative individual. Take a 48-year old man who says he feels as he didwhen he was 36; at the age of 36, if he had made the same type of response that he does now,he would have claimed to have felt as he did when he was 27; and so forth. This is an oddturn of events. How did the author get from the group results to reapplying the typicalresponse over and over to a single person? As (A) says, the author projects from the surveyresponse to hypothetical responses made by a single individual; he looks at the responses theindividual would have made at an earlier age, and then another earlier age, and so on, anduses that to show that the current response is ambiguous.

(B) The author never discusses what would have been the “most reasonable” thing for therespondents to say.

(C) The example of the 48-year-old man isn’t used as a counterexample; it doesn’t show thatpeople really didn’t “almost unanimously” respond by saying that they felt 75% of theirreal age. Instead, the example is used to show that the response, which the author neverdoubts, is inherently ambiguous.

(D) The author doesn’t set up two opposed statements in direct contradiction of each otherand force us to choose one or the other; he’s simply interested in showing that it’s difficultto understand what one statement means.

(E) The author never speaks of “manipulation” on the part of the questioners.

• In order to understand the author’s “techniques of reasoning,” you’ve got tounderstand what purpose the reasoning is intended to serve; i.e. what the author istrying to prove. Here the author’s purpose is stated in the third sentence: he wants toshow there’s a problem in understanding the response. The hypothetical case of theIncredible Regressing Man is intended to show that.

Page 39: 15-PrepTest 15 Expls - dl.keywin.orgdl.keywin.org/3/4/340700517960b9bee3575f6766ff5765.pdf · • Sometimes the LSAT writers throw you a real curve when a Reading Comp ... LSAT Test

© K A P L A N 37

SECTION III:

LOGICAL REASONING

Page 40: 15-PrepTest 15 Expls - dl.keywin.orgdl.keywin.org/3/4/340700517960b9bee3575f6766ff5765.pdf · • Sometimes the LSAT writers throw you a real curve when a Reading Comp ... LSAT Test

LSAT PREP ______________________________________________________________ LSAT Test XV Explained: Section III

38 © K A P L A N

1. (B)The stimulus begins with a claim made by Shakespeare-philes, who feel that in England, atleast, Shakespeare’s work has been known and loved by all classes, not just the elite. Thekey phrase comes midway: “Skepticism about this claim is borne out by. . . .” Thisindicates that the author’s main point is to take issue with that claim. Meanwhile, thecounterevidence comes from only one period and source, the bound copies ofShakespeare from the early 1800’s that the author feels could not have been available toordinary folk. So (B) is correct in taking issue with the original claim, and has anappropriately narrow scope (“at some time in the past”).

(A) The issue here is ‘Who historically has revered Shakespeare (the elite or all classes?),’and not ‘How can you tell the elite apart from everyone else?’

(C) What aspects? What lack of appreciation today?

(D) goes too far. The evidence, as noted above, comes only from the 1800’s. The authorcannot be leading to a blanket statement that only the educated upper class has ever knownand loved Shakespeare.

(E) It’s not the elite who are skeptical but the author, and that skepticism is of the claimmade in sentence 1, not of whether Shakespeare in fact wrote worthy plays.

• The main point or conclusion must follow logically from the given evidence. Payclose attention to the topic and scope in hacking your way through the answerchoices.

• Don’t confuse the author’s attitude (in this case, skepticism) for the attitude of acharacter or group that appears in the stimulus.

2. (E)Even if only the educated rich of the early 1800’s could own and peruse the books that theauthor cites, who’s to say that everybody else couldn’t have known, studied, and lovedShakespeare by seeing his plays performed? The unwarranted assumption that peoplecould only get to know Shakespeare’s plays in book form drives a stake through the heartof the logic.

(A),(B) The conclusion drawn, which lest we forget is (B) in Question 1, isn’t aesthetic—“literary quality,” to quote choice (A)—but sociological, on the topic of ‘Who has alwaysrevered Shakespeare?’ And contrary to (B), the evidence isn’t “purely economic”—itconnects money to awareness of Shakespeare.

(C) is way off. No 20th century standards are cited, and it’s not 18th century events that arebeing judged or evaluated but a contemporary claim about the nature of Shakespeare’sperennial fans.

(D), like (A) and (B), misunderstands the argument as an attempt to judge literary quality.

Page 41: 15-PrepTest 15 Expls - dl.keywin.orgdl.keywin.org/3/4/340700517960b9bee3575f6766ff5765.pdf · • Sometimes the LSAT writers throw you a real curve when a Reading Comp ... LSAT Test

LSAT PREP ______________________________________________________________ LSAT Test XV Explained: Section III

© K A P L A N 39

• Often when a stimulus generates two questions, the second is easier than the firstand should be attempted first. In this case, however, figuring out the author’s point(Q. 1) was almost certainly a prerequisite for determining the reasoning flaw (Q. 2). Infuture, survey both questions instead of making a knee-jerk decision as to which totackle first.

3. (C)If you recognize the president’s subtle but definite scope shift, you can knock this questionoff in a jiffy. All the evidence shows is that the material used in the mailings is recycled. Butthe conclusion is that the mailings are recyclable—which is different. The president mustbe assuming that that which has been recycled can be recycled again.

(A) The conclusion deals with the recyclability of the mailings only. Paper put to otheruse is outside the scope.

(B) The Kaplan “Denial Test” shows you why (B) is wrong. Suppose for the moment that(B) is false: Suppose some of the group’s mailings lack windows. Would that new factaffect the argument? Not a bit; the mailing materials could still be completely recyclable. Incontrast, note that if (C) is false—if the window material isn’t recyclable—then theargument falls apart.

(D) As anyone who has discarded newspapers or cans in the normal trash will attest, thatwhich is recyclable isn’t always recycled! The issue here is recyclability, which has nothingto do with what actually happens to the mailings when they arrive at people’s homes.

(E) The conclusion concerns only “these mailings”—the ones sent from HQ. That othermailings may or may not be sent from elsewhere is beside the point.

• Remember the Kaplan “Denial Test”: The test of a necessary assumption is that if thestatement is denied or proven false, the argument suffers damage.

• Always be on the lookout for scope shifts. They don’t only occur in arguments thatwe’re explicitly told are flawed. They can be committed any time, by any arguer.

4. (A)The first line of attack is to mentally rewrite the first sentence. (And paraphrasing a keyidea is always smart anyway.) If that “frequently expressed view. . . is false,” it must meanthat, to the author, written constitutions are inherently no more liberal than, and may beeven less liberal than, unwritten ones. Which is all (A) is saying. Notice that the actual“therefore” sentence, the one you expect to be the conclusion, does in fact simply echosentence 1: It asserts a condition (liberal interpretation and application) that’s necessary fora written constitution to become liberal, in contrast to the view that a written constitution isper se liberal.

Page 42: 15-PrepTest 15 Expls - dl.keywin.orgdl.keywin.org/3/4/340700517960b9bee3575f6766ff5765.pdf · • Sometimes the LSAT writers throw you a real curve when a Reading Comp ... LSAT Test

LSAT PREP ______________________________________________________________ LSAT Test XV Explained: Section III

40 © K A P L A N

(B) screws up the phrase “properly understood.” The author uses it in sentence 3 tointroduce her view of what a constitution should be properly understood to be—namely, asum of procedures. But (B) crazily implies that one cannot properly understand a writtenconstitution. Huh?

(C), besides making no sense (how could it be easier to misinterpret a written documentthan a set of concepts that are mental or verbal only?), falls into much the same trap as (B),by implying that the issue is how a constitution can be understood or interpreted, whenthe issue is, in fact, How liberal per se, is a written constitution?

(D) The preservation of constitutions is never mentioned or alluded to.

(E) There probably are criteria for evaluating how a constitution is to be interpreted andapplied, but the author never gets into that. What she’s presenting, in the last sentence, arecriteria for deeming a constitution as “liberal.”

• As noted above, paraphrasing an author’s key ideas is always smart.

• Don’t be surprised when the author seems to state her conclusion twice—once at theoutset and once at the end, albeit in slightly different words. This is a common tacticin argument and debate. The LSAT writers want to see whether you recognize therepetition or mistake the two sentences for unrelated thoughts.

5. (B)Sentence 2, by itself, yields the correct answer. (B)’s sentiment, that one cannot evaluate theliberalness of a written constitution from the writing alone, directly follows from sentence2’s assertion that words have to be put into effect or practice before they can be assessed.

(A) contradicts both (B) and the text. By the author’s definition, a paper with words on it ismeaningless until those words get turned into action.

(C) makes little sense (it has to be somewhat handy to have a written constitution to consultfrom time to time), and shifts the scope by implying that the issue is which type ofconstitution is preferable, when it’s really about which one is inherently more liberal.

(D) reflects a faulty paraphrase of sentence 1, and if you picked (D) please read and followthis explanation carefully. To state, as the author does, that it’s false that writtenconstitutions must be more liberal than unwritten ones, doesn’t mean that the reverse istrue—that unwritten ones must be more liberal than written ones. As noted in theexplanation to Question 4, all we can assume is that written ones don’t have an edge when itcomes to liberalness. In fact, as we see from the argument as a whole, the author believesthat neither type of constitution is inherently more liberal than the other; it all depends onhow a constitution is interpreted and applied.

Page 43: 15-PrepTest 15 Expls - dl.keywin.orgdl.keywin.org/3/4/340700517960b9bee3575f6766ff5765.pdf · • Sometimes the LSAT writers throw you a real curve when a Reading Comp ... LSAT Test

LSAT PREP ______________________________________________________________ LSAT Test XV Explained: Section III

© K A P L A N 41

(E), besides leaving out the issue of application (which is coequal with interpretation in theargument), commits the common error of mistaking a necessary condition for a sufficientone. Note that Keyword “only”! Just because a constitution is interpreted and appliedliberally doesn’t make it liberal—there may be other requirements that need to be metbefore a constitution is deemed to be a liberal one.

• Don’t take the question stem to mean that you have to combine all the stimulusstatements in order to come up with the right answer. More often than not, thecorrect inference either follows from one sentence (as it does here), or can be derivedby combining two sentences.

• You may have left this argument and its two questions successfully, and still notknown exactly what the author means by a “liberal constitution.” Who cares? Fullcomprehension of the content is less important than full command of the purposeand structure.

6. (E)The author recommends that scientists focus on Earth-like planets and carbon-based lifewhen estimating the probability of life on other planets, even though there could well benon-carbon-based life on planets unlike ours. How come? Because all known life forms arecarbon-based, and only Earth, to our knowledge, supports life. So the author issuggesting—for whatever reason—that we’re wisest to use what we do know in order tomake reliable predictions about what we don’t. Only (E) explicitly evokes this idea of“concentrate on the known.”

(A), as a principle, would support a recommendation that we not make use of observedphenomena when making inferences about unobserved phenomena. This is pretty muchthe antithesis of the author’s point of view.

(B) sets up a contrast—explaining all phenomena vs. explaining some phenomena—that hasno connection to the stimulus whatsoever.

(C), like (A), goes against the text. The author isn’t opposed to making estimates aboutextraterrestrial life, as is the principle in (C); she’s simply in favor of basing those estimateson known rather than unknown factors.

(D) may sound impressive, but is no closer to the text than (B) is. The author is in no sensetalking about “explaining phenomena,” and has nothing to say about reliance on a fewprinciples vs. reliance on many.

• Some LR arguments focus on scientific issues and terminology. If you’reuncomfortable with them, remember our advice about science passages in ReadingComp.: Don’t even try to read like a scientist. Deliberately “dumb down” yourreading: Paraphrase loosely, concentrate on the overall thrust of the text, and don’tget bogged down in details. Here, if you simply isolate the topic and scope as “howto estimate the likelihood of alien life,” (E) would have to jump out as the only choicethat even comes close.

Page 44: 15-PrepTest 15 Expls - dl.keywin.orgdl.keywin.org/3/4/340700517960b9bee3575f6766ff5765.pdf · • Sometimes the LSAT writers throw you a real curve when a Reading Comp ... LSAT Test

LSAT PREP ______________________________________________________________ LSAT Test XV Explained: Section III

42 © K A P L A N

7. (B)The politician begins by citing the need for the redistribution of wealth, without whicheconomic injustices will flourish. He then predicts that such injustice, if it peaks, will turnthose who are suffering to violence. Finally, he asserts that the nation must do whatever itcan to alleviate problems—inferably including the economic injustice just described—thatwould otherwise lead to violence. So the implicit point must be: We had betterredistribute wealth, choice (B), lest the chain of causation just described come to pass.

(A) Though the politician believes that violence is inevitable result of intolerant injustice,and probably believes that that is regrettable, his purpose is neither to justify nor tocondemn—his purpose is to recommend a course of action that will avoid a lot ofescalating problems.

(C) This can’t be the politician’s conclusion because no evidence to this effect is provided;the topic is economic justice. In any event, he apparently believes that decisions should bebased on practical realities and not on either of the grounds that (C) mentions.

(D) Actually, his point is that economic injustice had better be remedied in order toprevent intolerable social conditions that he believes are an inevitable consequence of itscontinuation.

(E), like (E) in Question 5, mistakes a necessary condition for a sufficient one. “Unless” is aKeyword signaling necessity. Redistributing wealth may not bring about economic justice,but without it, violence is inevitable.

• Inferences and conclusions share a key characteristic with assumptions: They mustbe true in order for the argument to work. Maintain a high standard of truth foranswer choices in all three question types.

• Make sure you fully understand the logic inherent in the word “unless.” In thestatement “Unless X (redistribute wealth), then Y (economic injustice)”, the X elementis necessary to avoid the Y element. That doesn’t mean that X is sufficient, by itself,for Y to be avoided; once again, there may be other factors involved. Think of thespecifics in this example in common sense terms: If the nation doesn’t redistributethe wealth, then there will definitely continue to be economic injustice. If the nationdoes redistribute the wealth, then that requirement is satisfied, and it’s possible toalleviate the economic injustice, but by no means is it a sure thing.

Page 45: 15-PrepTest 15 Expls - dl.keywin.orgdl.keywin.org/3/4/340700517960b9bee3575f6766ff5765.pdf · • Sometimes the LSAT writers throw you a real curve when a Reading Comp ... LSAT Test

LSAT PREP ______________________________________________________________ LSAT Test XV Explained: Section III

© K A P L A N 43

8. (A)The data aren’t presented in the most helpful order. You might have tried to rearrange thefacts chronologically, and made a mental picture of the situation, too: In 1985, they found38 of these beetles in a time span of two hours, while in 1989, in the same place, they onlyfound 10 in nine hours. Those data might suggest a decline in the beetle population overthose four years, but no: The author’s conclusion is that there probably was no drop because(Evidence Keyword) ‘85 was wet and ‘89 was dry. Sentence 1 tells us that these normallymotionless beetles move around more in wet years. What’s the connection between this factand the data presented? The implication is that in 1989, despite what the observer reported,there were pretty much the same number of beetles in that spot, but they weren’t moving asmuch so they couldn’t be seen. (A) supports the argument by shoring up thisimplication—to escape the gaze of the observer the beetles would have to be pretty darnedinvisible when motionless. It makes more plausible the notion that the observer missedapproximately 28 additional beetles that were actually there in 1989.

(B) doesn’t necessarily describe the environment at which the observer was conducting thestudy, and in any case, other beetle habitats are outside the scope of this argument, whichconcerns one locale only.

(C) mentions reproduction in the expectation that you’ll connect it with the issue at hand,which you shouldn’t: A comparison of the number of beetles in two separate years hasnothing directly to do with reproduction activity. But even if they were connected, (C)’ssuggestion that beetle movement relates to reproduction activity would weaken theauthor’s logic, because it would link the dry weather in 1989 to less frequent reproductionand hence support the idea of a 1989 population drop.

(D) Useless background information: So they’re rare, so what? The fact remains that somewere observed in 1985 and fewer in 1989, and the issue is the explanation for that decrease.(D)’s historical note is at most of footnote interest only.

(E) The reference to predators is tempting, if only because eaten beetles are nonexistentbeetles, but it’s a dead end. If predators are relevant in this context, then why would theybe any more or less relevant in 1989 than 1985? (E) completely ignores the essential timecomparison at the heart of the logic, so although you may feel that it raises a potentiallyrelevant element, it can’t possibly strengthen the reasoning.

• One element of paraphrasing that you may want to develop, is to rearrange anargument’s pieces of evidence so that you can better see the chain of logic or thechronology at work. Take some simple shorthand notes if it helps you—remembering to keep all scratchwork in the test booklet, of course.

• Don’t fall for tempting choices that appear to lead in a plausible direction, butultimately require an extra leap of faith. The issues of reproductive behavior (C) andpredators (E) may seem relevant at first glance, because these things can impact onbeetle population, but without more information and a stronger connection to theauthor’s conclusion, they fall short.

Page 46: 15-PrepTest 15 Expls - dl.keywin.orgdl.keywin.org/3/4/340700517960b9bee3575f6766ff5765.pdf · • Sometimes the LSAT writers throw you a real curve when a Reading Comp ... LSAT Test

LSAT PREP ______________________________________________________________ LSAT Test XV Explained: Section III

44 © K A P L A N

9. (B)In concluding that raising blood-magnesium levels can cure fatigue, the author interpretsthe observed correlations between fatigue (especially fatigue involved in chronic fatiguesyndrome) and low levels of blood magnesium as indicating that the low levels ofmagnesium caused the fatigue. But he hasn’t shown this to be the case. As (B) points out, it’squite possible that it’s fatigue that causes lower concentrations of magnesium in the blood.If that were the case, raising the level of blood magnesium would simply attack a result offatigue, not the cause, and would be unlikely to effect a cure.

(A) The argument doesn’t rely on the claim that malabsorption of magnesium is always thecause of low blood magnesium; the only causal link that plays a role in the conclusion isthe link between low blood magnesium and fatigue.

(C)’s possibility is irrelevant; the fact that there is some variation in magnesium levelsamong healthy people wouldn’t challenge the author’s belief that raising the levels ofblood magnesium would cure the fatigue associated with the syndrome.

(D) and (E) both criticize the author for not being specific enough, but there’s no reason heshould be specific. He’s discussing in the most general terms what sort of treatment wouldcure chronic fatigue; he’s not writing out a prescription. Since his conclusion is general, heneedn’t do the things discussed in (D) and (E).

• The mistaking of a correlation for a causal relationship is one of the most commonlogical flaws tested on the LSAT; as soon as you read “is invariably associated with,”you should have expected the author to make this mistake.

• There are some traditional ways that the correct answer exposes thecorrelation/causation flaw: Sometimes it names a third unsuspected cause for bothconditions; sometimes it points out that the causation may run the other way, as (B)does here.

• The second sentence, about the general correlation between malabsorption ofmagnesium into the blood and other types of fatigue, provides supportingbackground information, but is not really central to the conclusion; any choice thatdwells on “the absorption of magnesium into the bloodstream” is missing the point.

10. (B)The consumer advocate argues that explicit safety labels should be required for toysbecause it would enable parents to be more effective in preventing toy-related injuries; thelabels currently required, “age-range” labels, don’t do enough to reduce such injuries. (B)strengthens this argument by confirming the advocate’s claim that age-range labels andsafety labels have a different effect on parents. To take the author’s own example, whenparents see that a toy is recommended for ages three and up, they take this to mean thatyounger children won’t understand or enjoy the toy; they don’t realize that it might meanthat the toy is dangerous for children younger than three. If the label specifically warnedthat the toy can be dangerous, parents would presumably be doubly careful that it not fallinto their young children’s hands.

Page 47: 15-PrepTest 15 Expls - dl.keywin.orgdl.keywin.org/3/4/340700517960b9bee3575f6766ff5765.pdf · • Sometimes the LSAT writers throw you a real curve when a Reading Comp ... LSAT Test

LSAT PREP ______________________________________________________________ LSAT Test XV Explained: Section III

© K A P L A N 45

(A) says some toys aren’t dangerous; this doesn’t show the need for requiring safety labelsfor dangerous toys.

(C) doesn’t strengthen the advocate’s argument because the advocate wasn’t specificallyinterested in the case of children under three; rather, she was trying to demonstrate thatsafety labels are more effective than age-range labels in preventing toy-related injuries forchildren of any age.

(D) suggests that safety labels wouldn’t be very effective in reducing injuries, or at leastinjuries to those children whose parents don’t read labels.

(E) focuses on a particular hazard, but the advocate was speaking of toy-related injuries ingeneral; she never said that safety labels would be especially effective in preventingchoking.

• Beware of choices that focus on the author’s example. Most such choices simply seizeon an irrelevant aspect of the example and give it undeserved importance. Theexample of the “three and up” label is merely meant to explain what the advocatemeans by “age-range” labels; it’s explanatory, and hence very unlikely to be involvedin a strengthener.

11. (C)The whole stimulus is intended to prove the initial statement that the toy-labeling lawshould require explicit safety labels; the rest of the stimulus—the explanation of what toy-labeling laws currently require and why the proposed change would be animprovement—is intended to support that statement. Clearly, the statement in question isthe argument’s conclusion.

(A) As we’ve already said, the statement isn’t used to support a further conclusion, thestatement is the conclusion.

(B) There are no “conflicting” goals discussed, and the statement isn’t a compromise;instead, it’s the single goal for which the advocate argues throughout the stimulus.

(D) Huh?! Obviously, the advocate never attempts to refute her own conclusion!

(E) The statement isn’t a “particular instance,” it is itself the general principle underdiscussion; the only particular instance discussed is the “three and up” label, and that’s aninstance of age-range labeling.

• This argument reminds you that the conclusion can come anywhere in the argument.

• Every once in a while you get a gift. That’s why you should look at every question oneach section. That’s also why you should always look at both questions on a double-question set, even if you can’t answer the first; sometimes one of the questions ismuch easier than the other.

Page 48: 15-PrepTest 15 Expls - dl.keywin.orgdl.keywin.org/3/4/340700517960b9bee3575f6766ff5765.pdf · • Sometimes the LSAT writers throw you a real curve when a Reading Comp ... LSAT Test

LSAT PREP ______________________________________________________________ LSAT Test XV Explained: Section III

46 © K A P L A N

12. (E)The author says that organic farming leaves less land available to serve as habitat for localwildlife than chemical farming leaves, because organic farming requires that more acreagebe cultivated in order to produce the same amount of food. The author reasons that themore acreage cultivated by farmers, the less land is left as habitat for local wildlife. Hemust be assuming, as (E) says, that the land cultivated as acreage by organic farmers can nolonger serve as a habitat for wildlife; otherwise, the increase in land cultivated wouldn’tconstitute a loss of wildlife habitat.

(A) The author hasn’t discussed the health threat posed by chemical farming at all; for allwe know he accepts the argument that chemical farming threatens the health of wildlife.He’s merely making the counterpoint that organic farming also has a drawback as far aswildlife is concerned.

(B) is similar to (A); the author doesn’t imply or assume anything about whether or notchemicals cause harm to wildlife. His only point is that organic farming cultivates a widerarea, and thus reduces wildlife habitat.

(C) The author is only interested in the effects that the two farming methods have onwildlife, so he needn’t assume anything about their other disadvantages.

(D) The author compared the acreage needed for total food production; he doesn’t get intowhat type of crops are being produced, so he doesn’t really need to assume anything aboutthat.

• Keep the scope of the conclusion in mind. The conclusion only concerns thecomparative amount of wildlife habitat left by the two methods. The author needn’tassume anything about any other aspects of the two methods.

• A tricky type of wrong answer choice is one that introduces a distinction the authordoesn’t make, like (D). If the distinction doesn’t play a role in drawing theconclusion, it probably doesn’t play a role in any assumption. Our author doesn’tdiscuss what type of crops are grown; his argument is made in terms of “amount offood” and doesn’t require that the two methods of growing crops are equally fitted togrowing the exact same crops.

13. (A)The stimulus is pretty simple: Reptiles have these characteristics, therefore alligators—which we have to know or at least accept as being reptiles—have ‘em, too. In the same way,(A) argues that whatever’s true about green plants must be true about grass (which is asubset of green plants).

(B) “Some red butterflies” is more limited than the stimulus’ “[all] reptiles,” and one redbutterfly is not a class or subset of a larger group, as alligators are of reptiles or grass is ofgreen plants.

Page 49: 15-PrepTest 15 Expls - dl.keywin.orgdl.keywin.org/3/4/340700517960b9bee3575f6766ff5765.pdf · • Sometimes the LSAT writers throw you a real curve when a Reading Comp ... LSAT Test

LSAT PREP ______________________________________________________________ LSAT Test XV Explained: Section III

© K A P L A N 47

(C) would be parallel if it read, “All books provide knowledge about the empirical world,so Woolf’s book does likewise.” But as (C) is written, that books can provide suchknowledge does not imply that each individual book does so.

(D) The Ali film is not a subset of Fassbinder films, it’s one particular example. To beparallel, (D) might have read something like this: “All members of the Hanbury familyhave seen all of Fassbinder’s films, so Dierdre Hanbury has seen all of his films.” Note thatthere the author would be assuming that Dierdre is a member of that family, just as ouroriginal assumes (or knows) that alligators are reptiles.

(E) features two separate terms—running a risk and suffering a broken bone—in a way thatthe stimulus doesn’t. (E) would be closer if it read “Skiers run a high risk, so skier Lindsayruns a high risk.”

• Students often wonder about the difference between matters of “commonknowledge,” which you’re expected to have and use on the LSAT, and matters of“outside knowledge,” which you’re not. Question 13 is interesting in this regard. Youneed to know (or at least to accept without a qualm the author’s belief), that alligatorsare a subset of reptiles and grass is a subset of green plants in order to wade throughthis question without mishap. This is more detailed knowledge than the LSATusually calls for, and it may be that those examinees who complained about thisquestion in June of 1995 had a legitimate beef. On the other hand, none of the otherchoices features any kind of a subset relationship in the same way as the originaland (A), so it’s probably something we’re supposed to swallow without comment.(And don’t even get started on the speculation that some other colors of grass mightnot qualify as “green plants”..!)

14. (C)The author is trying to establish that waste of taxpayer dollars occurs at weapons-producing plants. As proof, he cites the fact that the government has decided to reopen aplant and allow it to violate environmental, health and safety laws, when it could just aseasily produce the weapons at a safer facility. Didn’t something get lost in the shuffle?What happened to taxpayer dollars? What does compliance with safety laws have to dowith wasting money? As (C) says, the evidence doesn’t address the issue of wastingtaxpayer money, which is the subject of the argument’s conclusion.

(A) The main problem with the reasoning isn’t that the author hasn’t backed up the claimthat the alternative site is safer, but that the issue of safety isn’t even the issue that theauthor set out to address.

(B) The author’s point that weapons laboratories are money wasters doesn’t undermine hisclaim that production plants are also money wasters.

(D)’s distinction between “research” and “weapons” is confusing; the research spoken of inthe argument is weapons research. The argument makes a distinction between weaponsresearch and weapons productions, and never confuses the two.

Page 50: 15-PrepTest 15 Expls - dl.keywin.orgdl.keywin.org/3/4/340700517960b9bee3575f6766ff5765.pdf · • Sometimes the LSAT writers throw you a real curve when a Reading Comp ... LSAT Test

LSAT PREP ______________________________________________________________ LSAT Test XV Explained: Section III

48 © K A P L A N

(E) The argument never compares research laboratories to production plants except to saythat they both waste taxpayer money; they don’t have to be similar to be compared in thatrespect.

• This is another example of an argument with different terms in the evidence and theconclusion. If your evidence is about compliance with health and safety laws, yourconclusion can’t be about wasting money. Don’t let the author off the hook! Don’tfigure: “Well, violating safety laws will lose money in the long run because ... “That’s an argument in itself, and it’s an argument the author doesn’t make.

15. (B)Dr. Godfrey notices that the more students work at their part time jobs, the worse they tendto do in school; he concludes that they’re doing badly because they’re working long hours.Dr. Nash also sees the correlation between part-time work and poor grades, but she drawsthe opposite conclusion; she believes the students who work long hours do so becausethey’re doing poorly in school. As (B) says, she offers an alternative interpretation for theassociation noticed by Godfrey.

(A) Dr. Nash disagrees with Dr. Godfrey’s claim that part-time work contributes tostudents’ academic problems; she never denies that those problems are serious.

(C) Dr. Nash accepts the accuracy of Dr. Godfrey’s evidence—i.e., she accepts that there isa correlation between students working long hours part time and doing poorly in school.

(D) Neither Dr. Nash nor Dr. Godfrey discusses the general question of whether schoolsare at fault for students’ academic problems; they discuss the much more narrow problemof whether the school policy of allowing part-time work has contributed to students’academic problems.

(E) Dr. Nash believes that there is a relationship between students’ academic problems andpart-time employment, just not the relationship that Dr. Godfrey espouses.

• This is yet another variation on the old correlation-for-causation-trick, as MaxwellSmart might say. The evidence gives a correlation between A and B; Godfreyconcludes that A causes B; Nash disagrees, and concludes that B causes A. Whenyou start recognizing this trick, such questions become a snap to answer.

16. (C)Since Dr. Godfrey says that students are having academic problems as a result of workinglong hours, and Dr. Nash says students are working long hours because they haveacademic problems, it would be nice to see which came first: the poor academicperformance or the long work hours. As (C) says, if we could look at the students inquestion and see whether they had academic problems before they started working, we’d bein a better position to know whether the work could have caused their problems.(Obviously, if the problems came before work, the work didn’t cause the problems).

Page 51: 15-PrepTest 15 Expls - dl.keywin.orgdl.keywin.org/3/4/340700517960b9bee3575f6766ff5765.pdf · • Sometimes the LSAT writers throw you a real curve when a Reading Comp ... LSAT Test

LSAT PREP ______________________________________________________________ LSAT Test XV Explained: Section III

© K A P L A N 49

(A) Neither Dr. Nash nor Dr. Godfrey said anything about the connection betweenacademic problems and future career success, so this question is irrelevant.

(B) Whether or not students also spend long hours at other activities doesn’t matter. Bothdisputants accept that there’s a connection between working long hours and doing poorlyin school; they only disagree about which condition causes the other, and (B) wouldn’thelp clear that up.

(D) hints at some undescribed skullduggery, but doesn’t address the question of whetherthey’re getting poor grades because they’re working a lot or vice versa.

(E), like (A), speaks of what happens after high-school; neither speaker says anything aboutthis, so the answer to the question posed in (E) wouldn’t help resolve the dispute.

• Take the time to unpack the question stem. You’re looking for a question whoseanswer would help you chose between Godfrey’s conclusion and Nash’s conclusion(This has some similarities to Section 1, Question 22). You can double-check a choiceby answering the question “yes” or “no” and seeing if it has an effect on thedisagreement.

• It’s important not only to know what the disputants disagree about, but also whatthey agree about, and what they don’t say anything about. Questions that center onthe latter two areas are irrelevant.

17. (A)X reasons that there is an “inevitable” trade-off between human and animal welfare inmedical research and that concern for human welfare should prevail. Y responds byproposing an alternative way to conduct medical research. Y does not believe that the onlyway to conduct medical research is on animals or humans. Y suggests that research beingdone on animals could be done in some other way (e.g., computer modeling), withoutcausing any suffering to animals or to humans. Y rejects X’s claim that there must be a“trade-off” between human and animal welfare; or, as (A) says, Y contradicts a premise(that there must be an inevitable trade-off) on which X’s argument relies.

(B) Y does not disagree with X about the weight to be given to animal suffering as opposedto human suffering. In fact, Y could even agree with X! If the only way to conduct medicalresearch involved humans and animals, Y may agree that human interests are moreimportant than animal interests. We just don’t know what Y believes about the relativeweight of human and animal interests because Y does not compare the two. All we know isthat Y doesn’t believe that there is an inevitable trade-off; Y believes that the same resultscan be achieved by other means without animal suffering.

(C) Y’s argument doesn’t present a logical consequence of X’s premises; it contradicts X’sargument by denying a premise on which X’s argument relies.

(D) Y disagrees with X’s argument; the new evidence about other types of research isintended to refute X’s point.

Page 52: 15-PrepTest 15 Expls - dl.keywin.orgdl.keywin.org/3/4/340700517960b9bee3575f6766ff5765.pdf · • Sometimes the LSAT writers throw you a real curve when a Reading Comp ... LSAT Test

LSAT PREP ______________________________________________________________ LSAT Test XV Explained: Section III

50 © K A P L A N

(E) Far from filling in a hole in X’s argument by supplying an assumption, or unstatedpremise, Y attacks an explicitly stated premise.

• If you have trouble with this type of question or if you are pressed for time, at leastmake sure you have a clear idea of how the second argument relates to the first:Agreeing? Disagreeing? Clarifying or refining? Here Y definitely disagrees with X,which means you can quickly strike out (E), (D), and also (C) which don’t disagree orcontradict X’s statement. This leaves a 50-50 chance of guessing correctly. Rememberthat intelligent guessing on test day will increase your score.

18. (A)Question 18 involves a scope shift. The evidence is that, in one particular environment,some types of bacteria experienced random mutations. The conclusion is that all geneticmutation is random. The credited response needs to link this specific example to thegeneral conclusion. Combine the stimulus with (A). The stimulus states that some geneticmutations are random, and (A) states that either all mutations are random or none are. Ifsome mutations are random, then all mutations must be random; the alternative optionisn’t possible. If (A) is true, all genetic mutations are random, which is our conclusion.

(B) At best, (B) only mildly supports the idea that bacteria mutations occur randomly; itdoesn’t however support the conclusion that all genetic mutation is random.

(C) Since the stimulus doesn’t tell us that all genetic mutation in bacteria is random (it onlymentions certain kinds of bacteria), answer choice (C) doesn’t allow us to concludeanything. (C)’s conclusion would only be true if all genetic mutations in bacteria wererandom.

(D) indicates that the genetic mutations of these particular kinds of bacteria would berandom whatever their environment, but doesn’t get us any closer to drawing theconclusion that all genetic mutation is random.

(E) too concentrates on the bacteria. (E) suggests that the experiment did a good job ofduplicating the bacteria’s behavior in nature, at least in some respects, but doesn’t allow usto draw conclusions about all genetic mutations.

• Be on the lookout for cases in which the author shifts the scope from “some” to “all.”The correct choice must move from specific evidence about some bacterial mutationsto a general conclusion about all mutations. (A) does this rather cleverly with ageneral statement that says, in effect, that whatever is true of some mutations is trueof all mutations (at least as far as randomness goes).

• In an assumption question, any answer choice that does not link the evidence to theconclusion (e.g., a choice in this question that never deals with anything beyondbacteria) is clearly wrong. Eliminate these answer choices quickly (e.g., (B) and (E)).

Page 53: 15-PrepTest 15 Expls - dl.keywin.orgdl.keywin.org/3/4/340700517960b9bee3575f6766ff5765.pdf · • Sometimes the LSAT writers throw you a real curve when a Reading Comp ... LSAT Test

LSAT PREP ______________________________________________________________ LSAT Test XV Explained: Section III

© K A P L A N 51

19. (A)According to the club rules, it’s necessary to be a member in good standing in order tovote. However, that doesn’t mean that being a member in good standing automaticallyentitles one to vote (is sufficient). Jeffrey is a member in good standing because he paid hisdues, but remember that being a member in good standing isn’t sufficient to guarantee hegets to vote. The club president could have disallowed his vote for another reason. Thomasmakes the common mistake of confusing that which is necessary with that which issufficient. To use the terminology in (A): Thomas fails to take into account the distinctionbetween something not being prohibited (Jeffrey is not prohibited from voting because heis a club member in good standing and only club members in good standing may vote)and its being authorized (every member in good standing automatically gets to vote).

(B) Thomas never attacks the president’s character but does focus on the question ofJeffrey’s eligibility to vote. Thus, neither part of (B) is correct.

(C) The only thing (C) could possibly be referring to is Thomas’ statement in the secondsentence “you’ve admitted that club rules say ...” But he’s not assuming that becauseAlthea didn’t deny what the club rules say that the club rules are true. He’s citing clubrules as independent evidence, and reminding his opponent that she’s already concededthis point.

(D) is irrelevant. The question is whether the President had the right to prevent Jeffrey, amember of good standing, from voting. Thomas doesn’t need to discuss what they werevoting about.

(E) is irrelevant. Thomas’ argument is about Jeffrey’s voting eligibility and doesn’t dependon whether Althea is an authority on club rules. Thomas’ argument doesn’t improve if sheis an authority and isn’t weaker if she isn’t.

• Often the flaw in a necessary vs. sufficient argument is easy to detect, so thetestmakers make the question more difficult by using unexpected phrasing in thecredited response. Don’t be surprised if the correct choice is a little puzzling. Beflexible in your approach to the choices; look for something that reflects the movefrom necessary evidence to sufficient conclusion.

• Remember that you don’t always need to use everything in the stimulus to answer aquestion. For instance, Althea’s reply simply serves as an illustration of Thomas’flaw. You can use Althea’s hint but don’t feel you have to work it into your thinking;if you can find the flaw easily without her help, that’s fine.

Page 54: 15-PrepTest 15 Expls - dl.keywin.orgdl.keywin.org/3/4/340700517960b9bee3575f6766ff5765.pdf · • Sometimes the LSAT writers throw you a real curve when a Reading Comp ... LSAT Test

LSAT PREP ______________________________________________________________ LSAT Test XV Explained: Section III

52 © K A P L A N

20. (B)People who consume calories beyond their proper weight-maintaining level normallystore fat and gain weight. But, people who regularly drink two or three alcoholic beverageper day (a subset of all people) and exceed their weight-maintaining caloric intake usuallydon’t gain weight. To resolve the discrepancy, we want the choice that most explains whythe general rule doesn’t apply to people who drink two to three alcoholic beverages a day.According to (B), when people drink two or three alcoholic beverages a day, the excesscalories that they consume tend to be “dissipated” as heat; so (B) explains why they don’tconvert those excess calories into fat and gain weight.

(A) says that some people who drink don’t exceed their weight-maintaining caloric intake,but we want to know about those drinkers who do exceed their weight-maintaining caloricintake.

(C) talks about people who do not drink alcoholic beverages and therefore doesn’t explainanything about people who do. Moreover, (C) doesn’t even speak of people who consumecalories beyond their weight-maintaining limit; (C) speaks of people who eat “high-caloriefoods,” a totally new idea.

(D) intensifies the mystery of the absence of weight gain in alcohol-drinkers withoutexplaining it.

(E) opens up a whole new question (fewer calories) to ponder, but doesn’t enlighten us asto why the alcohol drinkers mentioned in the conclusion don’t put on the weight.

• The testmakers have to phrase stimuli carefully so as to avoid ambiguity as much aspossible. This can work to your advantage; if you notice an odd turn of phrase, youcan be fairly sure that the precise meaning of the phrase has something to do withthe answer. Pay special attention to little words that imply some logical connectionbetween two things, like “thereby” in the stimulus to 20.

21. (D)The stimulus tells us that EEGs can “often, but not always” detect the abnormal electricalsignals associated with temporal lobe epilepsy. We’re given the first part of the conclusion:In general, a positive EEG reading reliably indicates temporal lobe epilepsy. Thiscorresponds nicely to the fact that EEGs can “often” detect the signals associated withepilepsy. But as the “although” indicates, the rest of the conclusion should correspond tothe other part of the evidence, the “not always.” So what can we conclude from the fact thatEEGs can’t always detect the abnormal electrical impulses? How about (D): a negativereading—failure to pick up evidence of abnormal impulses—doesn’t necessarily mean thattemporal lobe epilepsy is not present.

(A) is absurd; a positive reading can’t “reasonably reliably” indicate two contradictorythings.

(B) The stimulus only discusses temporal lobe epilepsy, so it would be inappropriate tointroduce other forms of epilepsy in the conclusion.

Page 55: 15-PrepTest 15 Expls - dl.keywin.orgdl.keywin.org/3/4/340700517960b9bee3575f6766ff5765.pdf · • Sometimes the LSAT writers throw you a real curve when a Reading Comp ... LSAT Test

LSAT PREP ______________________________________________________________ LSAT Test XV Explained: Section III

© K A P L A N 53

(C) The stimulus never mentions erroneous positive reading, only erroneous negativeones. Therefore, the correct conclusion can’t compare the relative accuracy of positive vs.negative readings.

(E) distorts the case; although a negative reading is not a sure indicator of the absence oftemporal lobe epilepsy, in no sense is a negative reading an indicator of the presence ofsuch epilepsy.

• Use Keywords. The “therefore” tells you that the last sentence is a conclusion. The“although” tells you that the missing clause will be a counterbalance to the statementabout the reliability of positive readings. This should remind you of the otheroccurrence of a counterbalance in the stimulus, in the phrase “often, but not always.”

22. (C)The quickest way to solve this Parallel Reasoning problem is to consider the “flawedreasoning” directly. The only way this conclusion must follow from the given evidence isif there’s roughly the same number of bicyclists in both age groups, and that’s a fact wecan’t take for granted. If there are, for instance, 100 bicyclists 18+ and 1,000 bicyclists under18, then 90 people with lights on their bikes would constitute a majority in the 18+category, whereas 100 people with lights on their bikes in the under 18 category would notconstitute a majority of that group. In this way, we can take care of the evidence in the firstsentence and still show how it doesn’t necessarily lead to the conclusion in the secondsentence. So the nature of the flaw is a confusion of proportion with number. Scanning thechoices, only (C) qualifies, with voters taking the role of bicyclists 18+, non-voters standingin for bicyclists under 18, and “being on the mailing list” the equivalent of “having lightson their bikes.”

(A) In the original, the majority in one group possesses a trait while the majority of anothergroup does not. (A), however, gives us the majority in the same group—people inSheldon—possessing different traits (buying gas on Monday and groceries on Tuesday).

(B)’s conclusion is a statement of causation (“the availability of videos was responsible”),which is nothing like the stimulus.

(D) can be rejected because its first piece of evidence is a statement about “every” memberof a group, and its conclusion evokes the concept of minimum (“must have at least two firetrucks”), neither of which is present in the stimulus. Also, note that the concept that isprevalent in both groups in the original, the aspect of majority, is only present in part in(D).

(E), like (D), veers away from the stimulus with a piece of evidence about “everyone” in aparticular group. Also (E) involves two traits or behaviors, knitting and sewing, whereasthe stimulus involves only one: having a light on one’s bike.

Page 56: 15-PrepTest 15 Expls - dl.keywin.orgdl.keywin.org/3/4/340700517960b9bee3575f6766ff5765.pdf · • Sometimes the LSAT writers throw you a real curve when a Reading Comp ... LSAT Test

LSAT PREP ______________________________________________________________ LSAT Test XV Explained: Section III

54 © K A P L A N

• Whenever the question stem announces the presence of a faulty argument, considerinvesting a few seconds to see whether you can spot the flaw directly. Between theargument at hand, and what you’ve learned about common LSAT logic flaws in yourKaplan course, the flaw may jump out at you—as it might have done here. And thenyou’re pretty much home free.

• When a conclusion is based on evidence about two separate groups, be on thelookout for the “numbers vs. percentages” scope shift.

23. (E)The author’s conclusion is that the government should not require removal of all asbestosinsulation because such removal disturbs asbestos, and when asbestos is disturbed itbecomes a health hazard. When left undisturbed, asbestos is harmless. (E) supports thisargument by suggesting a way in which asbestos removal may be worse for our health thanleaving the asbestos undisturbed. Not only does the initial removal pose a risk to ourhealth, but, according to (E), the removed asbestos remains a health hazard even afterremoval due to the risk that it will be disturbed again, resulting in the same negativeconsequences.

(A) The author is arguing that removing asbestos is more dangerous than leaving itundisturbed; the fact that there are other things more dangerous than asbestos is irrelevantto the argument and therefore doesn’t support it.

(B) doesn’t strengthen the argument because 1) we already know that the removal ofasbestos poses a health risk and 2) it implies that workers who wear their “required”protective gear can avoid the health risk of asbestos (which would weaken the argument).

(C) makes an irrelevant distinction between types of asbestos which doesn’t strengthen orweaken the author’s claim that it’s safer to leave asbestos alone (any kind of asbestos) thanto remove it.

(D) actually weakens the argument by suggesting that asbestos may be disturbed, andconsequently become a hazard, even if it is not deliberately removed; this weakens theauthor’s idea that it’s safe not to remove asbestos. If (D) were true, it wouldn’t be possibleto leave asbestos undisturbed; renovations or building demolition would inevitably andunintentionally disturb asbestos, thus posing a health risk even though the asbestos wasn’tremoved.

• Sometimes (but not often) the correct choice strengthens the argument in a stimulusby adding new evidence that supports the conclusion. This type of answer is almostimpossible to pre-phrase, since you’re unlikely to guess what new informationthey’ll introduce. However, you can most easily recognize useful new evidence if youhave a clear idea of how the author draws her conclusion. For instance, in thisquestion, you might summarize the argument as follows: It’s bad to remove asbestos,because removing asbestos causes disturbance and disturbance is bad. (E) mentionsadditional disturbance and thus should ring a bell.

Page 57: 15-PrepTest 15 Expls - dl.keywin.orgdl.keywin.org/3/4/340700517960b9bee3575f6766ff5765.pdf · • Sometimes the LSAT writers throw you a real curve when a Reading Comp ... LSAT Test

LSAT PREP ______________________________________________________________ LSAT Test XV Explained: Section III

© K A P L A N 55

24. (C)In a somewhat unusual argumentative structure (at least by LSAT standards), the authordraws two conclusions, both clearly marked by Keywords. The stem in 24 alerts us to therelevant conclusion for this one, namely: “Therefore, . . . the direction of the Earth’smagnetic field has changed over time.” The evidence for this conclusion is presented in thefirst two sentences: The direction of magnetization in solidified lava reflects the directionof the Earth’s magnetic field at the time the lava solidifies, and there are major differencesin this magnetization direction among solidified lava flows formed throughout the Earth’shistory. To show that this proves that the direction of the Earth’s magnetic field haschanged over time, the author must assume that once lava has solidified, the direction ofits magnetization is permanently fixed. If not—that is, if the direction of magnetization insolidified lava can change unpredictably—then there’s no way that the differingmagnetization directions documented in the evidence supports the notion that the Earth’smagnetic field has changed over time. The author must assume (C); otherwise, thedifferences noted could be the result of unpredictable changes within the solidified lavaflows, not changes in the Earth’s magnetic field at the time these lava flows were formed.

(A) The author doesn’t state that only lava can be used to measure the Earth’s magneticdirection, merely that it can be. Apply the Kaplan Denial Test: The argument doesn’t fallapart if other things besides lava can reflect the Earth’s magnetic field direction as it existedin the past, which confirms that (A) is not a necessary assumption.

(B) fails to mention magnetization at all! Both the evidence and conclusion deal withmagnetization, so any key assumption must include this concept. Instead, (B) introducesthe irrelevant concept of “consistencies.”

(D) includes an irrelevant comparison and, like (B), fails to mention magnetization. Applythe Kaplan Denial Test and it will become readily apparent that (D) is not a necessaryassumption.

(E) is outside the scope of the argument. Magnetized rocks? Hopefully, you dismissed thisanswer choice quickly.

• Not everything an author presents is relevant to the question asked. Use your criticalreading skills to evaluate the importance of the information presented. You can stillanswer Question 24 even if you remove the entire last sentence of the stimulus! Don’tget bogged down.

• Remember that the Kaplan 4-Step Approach to Logical Reasoning suggests that youmove directly to the answer choices if you are unable to pre-phrase an answer.Process of elimination works very well for this question because of the commonwrong answer types that appear.

• Learn to recognize the distortion common wrong answer type (as exhibited in choice(A)) so that you can dismiss these choices quickly on test day.

• (D) is another common wrong answer choice that appears on the exam; the irrelevantcomparison / distinction. Learn to recognize it and to eliminate it and you will garnermore points in less time on test day.

Page 58: 15-PrepTest 15 Expls - dl.keywin.orgdl.keywin.org/3/4/340700517960b9bee3575f6766ff5765.pdf · • Sometimes the LSAT writers throw you a real curve when a Reading Comp ... LSAT Test

LSAT PREP ______________________________________________________________ LSAT Test XV Explained: Section III

56 © K A P L A N

25. (D)Now we’re asked to weaken the author’s second conclusion (see the discussion for Question24 above). The author argues that lava flows that differ in age by several thousand yearshave roughly the same magnetic direction, but that lava flows that differ in age by muchmore time (hundreds of thousands or even millions of years) have different magneticdirections. Therefore, the author concludes, the Earth’s magnetic field changes slowly. Anyanswer choice that weakens this argument must undermine the connection between theslow passage of time and the slow change in direction. Choice (D), which states that themagnetic direction of lava can change rapidly (e.g., within two weeks of a single flow’ssolidification) undermines the argument that the Earth’s magnetic field, as represented bylava, changes slowly.

(A) doesn’t link the magnetic direction of lava to the Earth’s magnetic direction. Itintroduces the irrelevant idea of “iron-containing liquids.” Besides, who cares how themagnetic direction changes? The issue is whether it changes and how fast. Hopefully youdismissed (A) quickly as outside the scope of the argument.

(B) introduces useless background information. What’s happened since scientists beganmeasuring these magnetic directions in no way damages the link between the author’sevidence about the observation of past lava flows and the conclusion drawn from it.

(C) is consistent with the idea that the change occurred slowly (several time in a few millionyears). Remember that we want to weaken the argument.

(E) fails to mention magnetic direction at all, and instead focuses on an irrelevant aspect ofthe solidification process.

• Process of elimination can be a great strategy, especially in tough questions. If thecorrect answer is eluding you, focus on throwing out the ones that are obviouslywrong; i.e., the ones that correspond to Kaplan’s list of common wrong answerchoices.

Page 59: 15-PrepTest 15 Expls - dl.keywin.orgdl.keywin.org/3/4/340700517960b9bee3575f6766ff5765.pdf · • Sometimes the LSAT writers throw you a real curve when a Reading Comp ... LSAT Test

LSAT PREP ______________________________________________________________ LSAT Test XV Explained: Section III

© K A P L A N 57

26. (E)The key here is to notice that each statement is versed in formal logic. The tricky thing isthat the testmakers make use of equivalent phrases along the way. For example, “fall moreslowly” = “fall less rapidly.” It’s up to you to make the logical connection when it comes todifferent ways of saying the same thing. In short, it all comes back to critical reading.

So here’s the argument: If X (slower to adopt new tech than competitors), then Y (prodcosts fall more slowly than competitors). But if Y (prod costs fall less rapidly = prod costsfall more slowly), then Z (prices fall less rapidly). Choice (E) can be derived from using thecontrapositive on both of these statements: If NOT Z (prices fall AS or more rapidly ascompetitors), then NOT Y (prod costs fall AS or more rapidly). Continuing back to the firststatement, if NOT Y (prod costs fall AS rapidly), then NOT X (not slower to adopt new tech= as fast as competitors to adopt new tech). Put it together, it adds up to (E).

(A) The concept of “raising prices” is never mentioned in the original stimulus and is notthe opposite of “not lowering prices as rapidly.” The latter will definitely result in acountry being squeezed out of the global market. As for a consequence of the former(raising prices), we can’t infer a thing.

(B) Adopting technology slower than competitors will lead to being squeezed from theglobal market, but the author never states that this is the only way a company can besqueezed from the global market. There may be other ways a competitor gets squeezedfrom the global market (e.g., political instability in a country) that don’t involvetechnology. In other words, adopting technology slower than competitors is sufficient forbeing squeezed out of the global market, but we can’t infer, as (B) does, that it’s a necessarycondition.

(C) Did you catch the scope shift in this one? The mere adoption of new manufacturingtechniques is not the issue; it’s whether a country’s manufacturers’ adoption of saidtechniques (or technology) is faster or slower than that of its foreign competitors. (C) failsto compare how quickly the different countries adopt technology, and is thus notinferable.

(D) As seen above in (B), it’s possible for a group to be squeezed out of the market for areason other than differences in the rate of technological adoption.

• If you aren’t making progress on a tough question at the end of a section, your beststrategy may be to guess on it and devote the remaining time to revisiting otherpossibly easier questions you skipped earlier.

• When taking the contrapositive of a statement, be very careful when negating thevarious terms in the statements. For instance, the opposite of “slower to adopttechnology” is not “faster to adopt technology;” the opposite is “NOT slower toadopt technology.” Do you see the difference? This leaves open the possibility thatthe two countries adopt technology at the same rate, or as stated in the correctchoice, “as rapidly as.” This may seem like a minor difference, but understandingthis will help you see eye to eye with the correct choice.

Page 60: 15-PrepTest 15 Expls - dl.keywin.orgdl.keywin.org/3/4/340700517960b9bee3575f6766ff5765.pdf · • Sometimes the LSAT writers throw you a real curve when a Reading Comp ... LSAT Test

58 © K A P L A N

SECTION IV

LOGIC GAMES

Page 61: 15-PrepTest 15 Expls - dl.keywin.orgdl.keywin.org/3/4/340700517960b9bee3575f6766ff5765.pdf · • Sometimes the LSAT writers throw you a real curve when a Reading Comp ... LSAT Test

LSAT PREP ______________________________________________________________ LSAT Test XV Explained: Section IV

© K A P L A N 59

GAME 1 — Student Speeches(Q. 1-6)

The Action: Here’s a fairly standard sequencing game in which we’re asked to order thespeeches given by six students—H, J, K, R, S, and T. The Key Issues will involve ourtypical sequencing concerns:

1) Which speech is given in what time slot?2) Which speeches can, must, or cannot be given before what other speeches?3) Which speeches can, must, or cannot be given after what other speeches?

The Initial Setup: There are six speeches, so let’s write 1 through 6 across the page. (You’dprobably do exactly that if, in real life, you had to keep track of the students’ speeches.)Remember to list the students off to the side:

1 2 3 4 5 6H J K R S T

The Rules:

1) This rule is tough to symbolize, but think it through and jot it down as best you can.We’re told that the speeches given by H, J, and K can’t all be given consecutively. “NOHJK” is fine for shorthand, as long as you remember that these three entities can’t beconsecutive in any order. For example, KHJ and JKH would violate this rule too.

2) Likewise, the speeches given by R, S, and T can’t be consecutive. “NO RST” will sufficeas long as you realize that this rule has the same implications as Rule 1.

3) H’s speech is before S’s, so right away we know that H can’t go 6th and S can’t go 1st.You could include these implications in your master sketch, but they should beobvious enough that you need not even add them in. The important thing is to get therule down, so let’s shorthand it as “H . . . S.”

4) “NO J” over the 1 and 6 should do it for this rule.

5) T’s speech can’t be given consecutively with J’s speech. Let’s write “NO TJ or JT.”

Page 62: 15-PrepTest 15 Expls - dl.keywin.orgdl.keywin.org/3/4/340700517960b9bee3575f6766ff5765.pdf · • Sometimes the LSAT writers throw you a real curve when a Reading Comp ... LSAT Test

LSAT PREP ______________________________________________________________ LSAT Test XV Explained: Section IV

60 © K A P L A N

Key Deductions: See The Big Picture.

The Final Visualization: Here’s our master sketch and rules:

1 2 3 4 5 6H J K R S TNo HJKNo RSTH … SNo TJ or JT

No J No J

The Big Picture:

• Some games simply don’t have any earth-shattering Key Deductions. While youshould always attempt to combine the rules and seek out Key Deductions, you needto be able to recognize when a game is left fairly “wide open” as is the case here. Insuch cases, don’t knock your head against the wall—if you don’t see anything, savevaluable time by moving on to the questions.

• Even when there are no major Key Deductions, don’t assume that there’s nothing ofimportance to notice about the game before moving on to the questions. Remember,not all entities are created equal. To identify the most important entity, look for theone that appears in the most rules. Here, J is in two of the rules, and the game tendsto hinge on the placement of this entity. J can’t be in 1 or 6 (Rule 4), and J is alsoprohibited from being next to T (Rule 5). Some of the questions are bound to beanswered by T’s location limiting the possibilities for J’s placement. Whetherdirectly or indirectly, the restrictions placed on J, including J’s relation to T, will leadyou to the answer in quite a few of this game’s questions.

• Think before you draw. Make sure you understand what a rule says before you evenmake a stab at symbolizing it. In order to successfully master this game, you had tofully think through the implications of Rules 1 and 2 before putting pencil to paper.

Page 63: 15-PrepTest 15 Expls - dl.keywin.orgdl.keywin.org/3/4/340700517960b9bee3575f6766ff5765.pdf · • Sometimes the LSAT writers throw you a real curve when a Reading Comp ... LSAT Test

LSAT PREP ______________________________________________________________ LSAT Test XV Explained: Section IV

© K A P L A N 61

The Questions:

1. (D)Use Kaplan’s technique for acceptability questions to blow through this one in ten seconds.Compare each rule to each choice, and cross off any choice in violation of any rule. Rule 2is violated by (A). Rule 3 lets us axe (E). Rule 4 eliminates (B), and Rule 5 does away with(C) leaving us with (D).

• “Acceptability” questions are not only easy, but they also can help to clarify yourconception of the game.

2. (B)With T in 3, J can’t go in 2 or 4 (Rule 5). And J can’t be in 1 or 6 (Rule 4), so the only spaceleft for J is 5, choice (B).

(A), (C), (D), and (E) all could be true, but none must be true.

• Ask yourself the important questions. After placing T, the first thought in yourmind should have been, “how does this affect the most influential entity, J?”

3. (C)With S and T in 3 and 4, respectively, Rule 5 prohibits J from being next to T in 5. Ofcourse, 1 and 6 are off limits to J, so the only space open for J is 2. H has to be before S (Rule3), and the only open space before S is 1, so place H in 1. Since we can’t have STR in a row(Rule 2), R can’t go in 5, so K must go in 5 and R must go in 6. The entire order is complete,and only choice (C) corresponds.

(A), (B), (D), and (E) are all impossible.

• Work with the new information; get it down on paper and see where it leads. Takethe chain of reasoning as far as you can before moving on to the choices. In caseswhere you’re able to fill in every entity, there’s not a question in the world thetestmakers can ask that you can’t answer.

4. (A)With K in 1 and H in 5, Rule 3 forces S into slot 6. From there we’re left with J, R, and T tofill out spaces 2, 3 and 4. Rule 5 still forbids T and J from being next to each other, so one ofthem has to be in 2 and the other must be in space 4. R must separate J and T, so R must bein 3, choice (A).

(B), (C), (D), and (E) all could be true, but need not be depending on how you place T and Jin the 2 and 4 slots.

• If you hadn’t seen right away that T and J had to split the 2 and 4 spots, don’t worry.We were left with three students and three spaces for them, and if you tried placingeither J or T in 3, you would have seen that this would lead to a violation of Rule 5.From there it’s a short logical leap to recognizing that only R can be in 3.

Page 64: 15-PrepTest 15 Expls - dl.keywin.orgdl.keywin.org/3/4/340700517960b9bee3575f6766ff5765.pdf · • Sometimes the LSAT writers throw you a real curve when a Reading Comp ... LSAT Test

LSAT PREP ______________________________________________________________ LSAT Test XV Explained: Section IV

62 © K A P L A N

• Have confidence in your work! If you tried out choice (A) by trying to place Rsomewhere other than 3, and came to the conclusion that R indeed must deliver thethird speech, then there’s no reason to waste valuable time on the remaining choices.

5. (D)This question stem creates the block SRK. Rule 3 adds H to the mix (but not necessarilyright before S) to give us H . . . SRK. This block takes up four of our six spaces, leaving onlyJ and T. We’re plenty used to splitting up these two by now thanks to Rule 5, so we canfurther intuit that the H . . . SRK block can’t be in slots 1 through 4 or slots 3 through 6. Bothof these situations would force J and T next to each other. The block also can’t takeconsecutive slots 2 through 5, because that would leave J in either 1 or 6, which violatesRule 4. We know then that either J or T must be in between H and S with the other at oneend of the sequence. But once again we’re forced to remember that J can’t go at either end,so J must come between H and S, leaving T in either 1 or 6. So here are the onlypossibilities: THJSRK or HJSRKT. The only statement that could be true is choice (D).

• It took a lot of steps to get us to the two possibilities above. You didn’t need to takethe chain all the way in order to get the answer. Armed with H . . . SRK, you couldhave worked through the choices attempting to legally place the remaining J and T.Even with this method, you should be able to eliminate all but the correct choice.

• Look for blocks that take up space in the sketch. These make your work infinitelyeasier.

6. (A)After placing K in the 3 spot, there’s not much to do but try out each choice. If H is in 4, weknow that S must be in either 5 or 6. If S is in 5, the only spot open to J is 2. But if J is in 2,this gives us JKH which violates Rule 1. So S can’t be in 5. If S is in 6, the only spot open to Jis 5 (we just saw that J in 2 is no good in any case). But if J is in 5, we get KHJ consecutive,which also violates Rule 1. (A) is impossible and is therefore our answer.

(B) HSKJRT is an ordering that shows that this could be true.

(C) HJSKRT is an ordering that shows that this could be true.

(D) HTKSJR is an ordering that shows that this could be true.

(E) RJKTHS is an ordering that shows that this could be true.

• When you’re checking to see if something could happen, just try to make it work. Ifit works in even one case, then you’ve proven its validity.

• Check your previous scratchwork for help on questions like this. We can quicklyeliminate choice (E) thanks to correct choice (D) of acceptability Q. 1.

Page 65: 15-PrepTest 15 Expls - dl.keywin.orgdl.keywin.org/3/4/340700517960b9bee3575f6766ff5765.pdf · • Sometimes the LSAT writers throw you a real curve when a Reading Comp ... LSAT Test

LSAT PREP ______________________________________________________________ LSAT Test XV Explained: Section IV

© K A P L A N 63

GAME 2 — Radar Areas(Q. 7-13)

At first glance, this game may look odd and rather confusing, but don’t be intimidatedsimply because a game looks scary. Keep your composure; the first step, as always, is tonail down the game’s action:

The Action: Four circular radar areas—R, S, T, and U—are within the country of Zendu(although the specific country doesn’t matter, as you’ll see in the fifth bullet point underThe Big Picture). Four planes—J, K, L, and M—are each in the air over Zendu inaccordance with the rules. The rules are bound to clear the situation up for us, but even atfirst glance you should be able to intuit the Key Issues that the form the basis of the game:

1) Which planes are in which radar areas?2) Which planes can, must, or cannot be in the same radar areas as which other planes?

The Initial Setup: An Initial Setup hinges mostly on the rules built into the openingparagraph. Part (not all) of area R intersects area T, so draw a circle for R and another circleT that partially overlaps R. Next, we’re told that part of area S intersects T. However, wecan’t add S to our sketch yet, because we don’t know about S’s relationship to R; Ah, butnow we do: The very next clause in that sentence says that R and S don’t intersect eachother. So simply draw a circle for S that intersects T but not R. Finally, we’re told that areaU is completely within areas R and T. U must therefore fall in the space where R and Tintersect, so draw another circle in this space and label it U. Be careful not to falselyassume that U takes up the entire area where R and T intersect—this need not be the case.It’s quite possible for a plane to be in the area where R and T intersect, yet still outside ofarea U. This is more work than we’re used to for creating an Initial Setup, but in this game,understanding and visually representing the Initial Setup is half the battle.

J K L M

R U T S

Page 66: 15-PrepTest 15 Expls - dl.keywin.orgdl.keywin.org/3/4/340700517960b9bee3575f6766ff5765.pdf · • Sometimes the LSAT writers throw you a real curve when a Reading Comp ... LSAT Test

LSAT PREP ______________________________________________________________ LSAT Test XV Explained: Section IV

64 © K A P L A N

The Rules:

The “rules” in the intro paragraph allowed us to construct a picture of the physical layoutof the radar zones. The following indented rules tell us about the radar zones in which theplanes appear:

1) Our basic “loophole closer”: Each plane is in the game and somewhere within the radarareas of our sketch.

2) Pretty straightforward, but a little thought will let you take it one step further: If J is inarea S, then it’s either in the part of S that intersects T or it’s in the part of S that doesn’tintersect T. As long as you can visualize this, you can shorthand it any way you want. Onepossibility is to write a “J” in area S with one arrow pointing to the space where S and Tintersect and another arrow pointing to the rest of S that doesn’t intersect T.

3) “K ≠ J” should keep this rule nice and clear. Whatever area one of these planes is in, theother must be somewhere else. It would be difficult not to automatically combine this withthe previous rule to deduce right here and now that K is not in area S.

4) “L ≠ M” should do it for this one. Since we have no info so far on either of these planes,we can’t take this any further, so jot it down and move on.

5) M is in exactly one of the areas: M = 1 should suffice to help us remember this.

Key Deductions: Above, we combined Rules 2 and 3 to deduce that K cannot be in areaS.Now let’s look more closely at Rule 5. Since M is in exactly one area, M cannot be in areaU. Why? Because If a plane is in area U, then it’s actually in three areas: U, R, and T. Can Mbe in area R? Sure, as long as it’s not in the part of R that intersects with area T. The same istrue for M in areas T or S; it’s okay, as long as M isn’t in the intersecting part.

The Final Visualization: So, here’s our final, rather circular, sketch:

J K L MK ≠ JL ≠ MM = 1K not in SM not in U

R U T SJ

Page 67: 15-PrepTest 15 Expls - dl.keywin.orgdl.keywin.org/3/4/340700517960b9bee3575f6766ff5765.pdf · • Sometimes the LSAT writers throw you a real curve when a Reading Comp ... LSAT Test

LSAT PREP ______________________________________________________________ LSAT Test XV Explained: Section IV

© K A P L A N 65

The Big Picture:

• Force yourself to ask the relevant questions. When you’re told that S and T intersect,you must go further and ask “But wait, what about S and R? How do theyinterrelate?” This is the kind of active thinking that the Games section requires andrewards.

• Remember, critical reading isn’t a requirement limited to the Logical Reasoning andReading Comp sections—it’s vital in Logic Games as well, especially wheninterpreting rules. For example, it’s extremely important that you recognize that justbecause area U is in the part of areas R and T that intersect, U doesn’t necessarilyhave to be that entire area. Also, be sure that you didn’t get the rule backwards andput all of area R inside of area U.

• Don’t be thrown just because a game is unfamiliar. On some Games sections, thetestmakers throw in a somewhat unfamiliar game. While it won’t necessarily be thehardest game on the section, they do want to see whether or not you can think onyour feet, use your common sense, and not be thrown for a loop just because thesetup is slightly different from a more typical game.

• If you’re having a hard time getting a handle on a seemingly unfamiliar game, try outdifferent situations. For example, try putting M in areas R, S, T, and U and see whathappens. Mentally work out ways of separating L and M, or separating K and J. Thisway you can quickly get a feel for which planes can and can’t go where. Working outa few “what if?” scenarios should make you feel more comfortable with the gameand help you get a better handle on it.

• Don’t obsess over extraneous information. In this game, the fact that the zones fallwithin the country of Zendu doesn’t matter in the slightest. Did you draw a hugecircle representing Zendu and then try to put the radar areas inside? Why? Did youworry about the planes being in the air at exactly noon? Why? The time frame neverchanges; if it did, this would be more like a “process” game. Sometimes thetestmakers throw in this extra useless information in order to get nervous test takersto waste as much time as possible. Don’t fall for it.

Page 68: 15-PrepTest 15 Expls - dl.keywin.orgdl.keywin.org/3/4/340700517960b9bee3575f6766ff5765.pdf · • Sometimes the LSAT writers throw you a real curve when a Reading Comp ... LSAT Test

LSAT PREP ______________________________________________________________ LSAT Test XV Explained: Section IV

66 © K A P L A N

The Questions:

7. (D)As with all acceptability questions, let’s just check the rules against the choices. Rule 1 kills(A) because K isn’t included. Rule 2 takes care of (B). Rule 3 knocks off (E). (C) is the onlytricky one. Our Big Deduction above allows us to cross it off because we know that M,which can only be in one area, can’t be in area U, which is actually within two other areas. Ifthis wasn’t enough to kill (C), since area U is within area T, this choice places L and M inthe same area (T), which violates Rule 4. In any case, the only choice left is (D), which mustbe the acceptable listing.

• This is an unusual way of presenting an acceptability question. But your experiencewith acceptability questions in many other games should have helped you recognizethis one as such as soon as you saw the nature of the answer choices.

8. (A)If K is in 2 of the areas, then K must be in the area where R and T intersect (we alreadydeduced that K can’t be in S). In order for this to be possible, J must be in the part of S thatdoesn’t intersect T, since Rule 3 forces K and J into different areas. (A) can’t be true.

(B) and (C) must be true.

(D) and (E) could be true, but need not be.

• The relevant question here is: “How can K be within exactly two areas?” Thisquestion should lead you to a greater understanding of the game—the only way aplane can be in exactly two areas is if it’s in S and T or in R and T (U is no good,because any plane in U is automatically in three areas).

9. (E)We just saw in the setup for question 8 that K could be in area T, which quickly kills (A),(B), and (C). The only difference between (D) and (E) is whether or not J can be in area T.And lo and behold, we see that J can be in T from the correct choice (D) to acceptabilityquestion 7. Basically, there’s no reason why any of the planes cannot be in area T, and theanswer is (E).

• This is the second game in a row where the acceptability question helped on asubsequent question. When a question asks about what could possibly happen,check all of your previous scratchwork, especially the correct answer to theacceptability question.

Page 69: 15-PrepTest 15 Expls - dl.keywin.orgdl.keywin.org/3/4/340700517960b9bee3575f6766ff5765.pdf · • Sometimes the LSAT writers throw you a real curve when a Reading Comp ... LSAT Test

LSAT PREP ______________________________________________________________ LSAT Test XV Explained: Section IV

© K A P L A N 67

10. (E)Here’s our Big Deduction staring us right in the face: M in area U? Can’t do it; M can onlybe in one area, but U is within two others. Choice (E) is the five-second answer. If youhadn’t made this deduction up front, or simply forgot about it, you could get the answerby trying out the choices (which will no doubt take longer than five seconds).

(A) is possible; we just saw from Q. 9 that everyone could be in T.

(B) K can be in U as long as J isn’t in T.

(C) is possible as long as M isn’t in R. You may also have noticed that L was in R in one ofthe wrong (and therefore possible) answer choices from question 8.

(D) is possible; once again we benefit from turning to correct choice (D) for acceptabilityQ. 7 which validly places M in R.

• Who said this game was a killer? We’re more than half way through it and we’vealready seen some very easy questions (easy, that is, if you did the right work upfront).

• Don’t forget about your Key Deductions! If a non-if questions simply asks forsomething that cannot be true, ask yourself, “did I notice something that can’t betrue?” If so , scan the choices for it, and you’ve got yourself a five-second point—anice little reward for the work you put in up front.

11. (D)A single plane can’t be in what two areas? We know that S and R don’t intersect, so a planecouldn’t be in both S and R. Not surprisingly, that’s too obvious, and therefore not achoice. But we need go only one step further: Since area U is completely within area R,there’s no way for a plane to be simultaneously in areas S and U. As easy as that, choice (D).

(A), (C) The areas listed in each of these choices intersect, so it’s definitely possible for aplane to be in both areas at once.

(B), (E) Area U is within areas R and T.

• Actively pursue Logic Games answers. If you can, try to figure out the answer aheadof time and then look for it among the choices.

• Decode tricky question stems. The only possible difficulty with this question is inunderstanding what they’re looking for. When you need to, translate the questionstem into something more manageable, much like we did above.

Page 70: 15-PrepTest 15 Expls - dl.keywin.orgdl.keywin.org/3/4/340700517960b9bee3575f6766ff5765.pdf · • Sometimes the LSAT writers throw you a real curve when a Reading Comp ... LSAT Test

LSAT PREP ______________________________________________________________ LSAT Test XV Explained: Section IV

68 © K A P L A N

12. (E)M is in area T. Since M is in exactly one area, so we know that M is in the part of T thatdoesn’t intersect with R or S. Rule 4 says that L and M can’t be in the same area, so we knowthat L can’t be in the parts of R or S that intersect with T. Thus L can’t be in U; L must be inthe non-intersecting part of area R or the non-intersecting part of area S. At most, L can bein only one area, which leads us to correct choice (E).

(A) through (D) are each quite possible.

• When given information about an entity, search for the other entities most affectedby this information. The only relationship M has with another plane is with L, and Lshows up in three out of the five answer choices. These are good clues that signalthat the restricted entity we’re looking for will probably be L, so you may have beenable to save some time by recognizing this and checking out choices (B), (C) and (E)first.

13. (A)The only way for a plane to be within three areas is to be in area U, so L is in U (andtherefore is also in R and T). Rule 4 prohibits L and M from being in the same area, so theonly area open to M is S (which kills choices (D) and (E)). We deduced way back that Kcan’t be in area S, so axe (C). As we said before, the only way to be in three areas is to be inarea U. Since S and U don’t intersect, J can’t be in three areas. Cross off (B), which leaves uswith our answer, (A).

• Trying out choices could be time consuming, but by working with the newinformation and crossing off choices as you go along, the time required can usuallybe greatly reduced. Here, you may have tried out (A) first, and found it to beperfectly plausible; in that case, you needn’t even waste time on the other choices.

Page 71: 15-PrepTest 15 Expls - dl.keywin.orgdl.keywin.org/3/4/340700517960b9bee3575f6766ff5765.pdf · • Sometimes the LSAT writers throw you a real curve when a Reading Comp ... LSAT Test

LSAT PREP ______________________________________________________________ LSAT Test XV Explained: Section IV

© K A P L A N 69

GAME 3 — Car Pool Drivers(Q. 14-19)

The Action: Sequencing with one small twist: We’re asked to schedule four people—Fritz,Gina, Helen, and Jerry—on the days on which they drive, Monday through Saturday.Here’s the minor twist—four drivers for six days means that some folks have to drive onmore than one day. The Key Issues will be:

1) Who drives on which day?2) Which people can, must, or cannot drive on consecutive days?3) Which people drive before and after which other drivers?

The Initial Setup: Keep this simple; list the days across the page and the drivers off to theside, like so:

M T W Th F SF G H J

The Rules:

1) This is a loop-hole closer, but unlike most loop-hole closers, it’s an important one.Every person drives at least one day. This means that no driver is left out of the schedule(which until this rule was a distinct possibility).

2) “NO 2 IN A ROW” should do it for this one. For example, if Gina drives on Monday, shecan’t drive on Tuesday.

3) “NO F” over our M for Monday should remind us of this.

4) Break this rule up. “J” above the sketch with an arrow pointing to the “W” forWednesday and the “S” for Saturday will take care of the first part of this rule. However,Jerry could drive on both Wednesday and Saturday as well as driving on other days. Addsomething to your sketch to keep this added information in mind. We’ll write “at least”next to the J in our sketch to remind us of this, but whatever way you came up with,including just memorizing the rule, is fine as long as you come away with the correctinterpretation.

5) “If G drives on Mon, J isn’t on Sat.” Remember to jot down the contrapositive: “If Jdrives on Sat, G doesn’t drive on Mon.”

Page 72: 15-PrepTest 15 Expls - dl.keywin.orgdl.keywin.org/3/4/340700517960b9bee3575f6766ff5765.pdf · • Sometimes the LSAT writers throw you a real curve when a Reading Comp ... LSAT Test

LSAT PREP ______________________________________________________________ LSAT Test XV Explained: Section IV

70 © K A P L A N

Key Deductions: As with Game 1, no major deductions leap off the page at us. However, Jis in Rules 4 and 5, so let’s take a closer look there. J must drive on Wednesday, Saturday,or both (and possibly elsewhere, remember). If the conditional in Rule 5 takes place (i.e., Gdrives on Monday), then J can’t be on Saturday and would have to drive on Wednesday (atleast). Also think about the “NO 2 IN A ROW” rule. If we have J on Wednesday, then weknow that J can’t drive on Tuesday or Thursday. Since this line of thinking is predicated ona conditional, it won’t play out in every question. Therefore, it isn’t absolutely necessary tothink all this through in order to succeed in this game. However, noticing such things upfront is always helpful in the long run, and will help you to develop and to reinforce thegood habits and analytical thinking skills that will be rewarded on test day.

The Final Visualization: Here’s our final sketch:

M T W Th F SF G H JNo 2 in a rowIf G on Mon, J not on SatIf J on Sat, G not on Mon

No Fat leastJ

The Big Picture:

• Rule 4 is wordy and chock full of implications. Often, a rule like this would mandatethat Jerry drive on Wednesday or Saturday, but not both. However, the testmakersthrew us a slight curve here. No biggie—whenever you see a rule that’s slightlydifferent from what you’re used to, take a few seconds to think it through.

• Often, the entity that appears in the most rules will be the game’s most influentialplayer. Never let this entity stray too far from your mind.

• If an entity is limited to one of two positions, it’s often worth plotting out the resultsof each. We call this investigating the “limited options.” But it’s also important torecognize when this strategy doesn’t apply: Here, we could look at what happens ifJerry drives on Wednesday and then what happens when he drives on Saturday, butsince he can drive on both days, and on other days as well, discovering all of thepossible options that result from Jerry’s situation would be a near impossibility. Asstated in the Key Deductions paragraph above, mentally manipulating a fewpossible scenarios is probably all that’s necessary before moving on to thequestions.

Page 73: 15-PrepTest 15 Expls - dl.keywin.orgdl.keywin.org/3/4/340700517960b9bee3575f6766ff5765.pdf · • Sometimes the LSAT writers throw you a real curve when a Reading Comp ... LSAT Test

LSAT PREP ______________________________________________________________ LSAT Test XV Explained: Section IV

© K A P L A N 71

The Questions:

14. (D)This game too begins with an acceptability question. Pick a rule and compare. Rule 1 axes(E) since Fritz isn’t included. Rule 2 does away with (A). Rule 4 kills (C). Rule 5 takes careof (B), leaving us with (D).

• “Could be the schedule” is just another way of asking for “an acceptable schedule.”

15. (E)Not much to do except try out the choices. We’ll get to why the others are wrong, but first,if the order is G, F, J, H, F, G we see that (E) is possible and is our answer.

(A) This is a direct violation of Rule 4. We know that Jerry drives on at least one of thesedays.

(B) If Gina drives on Monday and Wednesday, then Jerry can’t drive on Saturday (Rule 5),which once again violates Rule 4.

(C) We know that Jerry drives on at least Wednesday or Saturday, so if he also drives onTuesday and Friday, this would have him driving two days in a row. That’s a no-noaccording to Rule 2.

(D) If Gina drives on Monday, then Rule 5 forces Jerry to drive on Wednesday (at least). IfJerry also drives on Thursday, he would be driving two days in a row, which again violatesRule 2.

• Notice how every wrong choice can be eliminated by paying attention to the Jerrysituation. Keep your most influential entities in mind.

• A non-if could be true question pretty much forces you to try out the choices. If itlooks like this will be time consuming, leave it temporarily and look for surerground, like Qs. 16 and 17 which give us more definite information. However, don’timmediately skip a question like 15; many non-if questions like this turn out to befairly straightforward, especially if you’ve thought the game through in the setupstage.

16. (C)If Jerry drives on Wednesday and Saturday only, then the contrapositive of Rule 5 tells usthat Gina can’t drive on Monday. Rule 3 already axed Fritz from Monday, and since in thisone Jerry drives only on Wednesday and Saturday, Jerry can’t drive on Monday either. Theonly person left for Monday is Helen, choice (C).

(A), (B), (D), and (E) could be true, but need not be.

• Always keep track of the entities in a game. Very often answers come from simplyrunning out of possible players to stick into a certain space.

Page 74: 15-PrepTest 15 Expls - dl.keywin.orgdl.keywin.org/3/4/340700517960b9bee3575f6766ff5765.pdf · • Sometimes the LSAT writers throw you a real curve when a Reading Comp ... LSAT Test

LSAT PREP ______________________________________________________________ LSAT Test XV Explained: Section IV

72 © K A P L A N

17. (A)If Gina drives on Monday and Saturday only, then Jerry must drive on Wednesday (atleast) according to Rule 4. Fritz and Helen must each drive on at least one of the remainingthree days. This leaves one day open that Fritz, Helen, or Jerry will drive on. Check thechoices, and we see that (A) must be true. Whoever drives on that extra day will be the oneother person besides Gina to drive on exactly two days.

(B) No, the person who drives on Wednesday (i.e. Jerry) could drive on Friday.

(C), (D) The possible ordering G H J F J G kills both of these choices.

(E) No, Fritz could be the driver on Tuesday (e.g. G F J H J G).

• Once again, being familiar with the numbers aspect of the game proved to be veryuseful. Here, remembering that all of the entities had to be used somewhere was halfthe battle.

18. (B)Nothing to do but check out the choices. If Gina drives on Monday, Jerry can’t driveSaturday (Rule 5), which means that Jerry must drive on Wednesday. According to Rule 2,he can’t drive on Tuesday as well. (B) can’t be true and is the answer.

(A) H F J F G J is fine, so (A) could be true.

(C) and (E) G H J F J G shows that both of these could be true.

(D) H J G F G J is acceptable, so (D) is not the answer.

• Don’t be surprised late in a game when questions begin to test the same concepts.This game should be renamed “Car Pool Drivers: The Placement of Jerry.” As soon asyou got to (B), your reaction should have been “seen it, done it, been there”—this isthe one that’s impossible.

Page 75: 15-PrepTest 15 Expls - dl.keywin.orgdl.keywin.org/3/4/340700517960b9bee3575f6766ff5765.pdf · • Sometimes the LSAT writers throw you a real curve when a Reading Comp ... LSAT Test

LSAT PREP ______________________________________________________________ LSAT Test XV Explained: Section IV

© K A P L A N 73

19. (E)Fritz drives twice, but the question stem and Rule 3 cut his options down to Thursday,Friday, and Saturday. The only way to get two days of driving out of him is for him todrive Thursday and Saturday (otherwise, he’d have to drive two days in a row, violatingRule 2). Since Saturday is taken, Rule 4 forces Jerry to drive on Wednesday, at least. Jerrycould also drive on Friday, with Gina on Monday and Helen on Tuesday, so (E) could betrue and is our answer.

(A) Fritz can’t drive three times since the stem says that he drives exactly twice. If any of theothers drive three times, that wouldn’t leave enough days for everyone to take a turndriving.

(B) Fritz drives twice, so that leaves four days for the other three people. No, three peoplecan’t drive exactly once; one of them must drive twice to cover all six days.

(C) Jerry must drive on Wednesday which is immediately before Fritz’s day on Thursday.

(D) No, Jerry must drive on Wednesday.

• Work with the given information and take it as far as you can. When you go to checkthe answer choices, the wrong ones will fall that much faster.

• Don’t be thrown by number-related choices like (A) and (B). These aren’t concernedwith who drives on what day, but rather with how many times each person drives.Continue to ask yourself relevant questions, which in this case would be: “How canwe cover all six days given the Fritz info in the stem?”

Page 76: 15-PrepTest 15 Expls - dl.keywin.orgdl.keywin.org/3/4/340700517960b9bee3575f6766ff5765.pdf · • Sometimes the LSAT writers throw you a real curve when a Reading Comp ... LSAT Test

LSAT PREP ______________________________________________________________ LSAT Test XV Explained: Section IV

74 © K A P L A N

GAME 4 — Plumber Teams(Q. 20-24)

The Action: This game involves distributing plumbers into four groups of two, whichsuggests a grouping distribution action. Technically, there’s a selection element as well:We need to select eight of the nine plumbers. However, it’s probably easier to think interms of having to leave one plumber out as opposed to having to choose eight of the nine.The plumbers are chosen from five experienced plumbers—F, G, J, K, and M—and fourinexperienced plumbers—r, s, t, and v. Let’s use capital letters for the experiencedplumbers and lowercase for the inexperienced ones. The Key Issues will be:

1) Which two plumbers are assigned to each team?2) Which plumbers can, must, or cannot be assigned to the same team?

The Initial Setup: We have four teams of two, so let’s put down four sets of two dashes.List the entities off to the side:

__ __ / __ __ / __ __ / __ __

EXFGJKM

inex.rstn

The Rules:

1) Basically a loop-hole closer, but the game would be undoable without it.

2) This rule narrows down the possible pairings enormously. Let’s write an “X” for“experienced” under one of the dashes in each of the four teams.

3) Three rules for the price of one: “NO FM”—“NO Fr”—“NO Fv.” These three restrictionstell us quite a bit about Frank’s situation.

4) “If t, then G or K” tells us that only these two experienced plumbers may team up withthe inexperienced Tim.

5) One rule for the price of one: “NO Jr” will do it.

Page 77: 15-PrepTest 15 Expls - dl.keywin.orgdl.keywin.org/3/4/340700517960b9bee3575f6766ff5765.pdf · • Sometimes the LSAT writers throw you a real curve when a Reading Comp ... LSAT Test

LSAT PREP ______________________________________________________________ LSAT Test XV Explained: Section IV

© K A P L A N 75

Key Deductions: Rule 3 severely limits Frank’s options, and Rule 4 limits it further. If Timmay only be teamed with Gene or Kathy, then obviously Tim may not team up with Frank.Add “NO Ft” to your list of Frank rules. This means that Sally is the only inexperiencedplumber that Frank may team up with. Does that mean “always Fs?” No; remember thatwe have five experienced plumbers and need only four of them. It is possible for Frank toteam up with another experienced plumber. If you like, you can write “If F, then Fs or F + X(experienced).”

The Final Visualization: Here’s what we’re armed with heading into the questions:

__ __ / __ __ / __ __ / __ __X X X X

EXFGJKM

inex.rstn

No FMNo FrNo FnIf t, then G or KNo JrNo FtIf F, then Fs or F + X

The Big Picture:

• Always work out a game’s number situation in advance, by asking yourself therelevant questions. In this case: “How can I split five experienced and fourinexperienced plumbers into four groups of two?” Answer: Always leave one out.Then, when you get more information, expand on what you know. As a consequenceof Rule 2, you should ask yourself: “How do the numbers work now?” Answer: Wemust include four experienced and four inexperienced plumbers OR fiveexperienced plumbers (giving us one team made up only of experienced plumbers)and three inexperienced ones.

• In three of the four games on this section it’s beneficial to identify the mostimportant entity, which is normally the entity that appears in the most rules. Here it’sobviously Frank. Once again, there are no earth-shattering deductions to be made inthis game, but thinking through Frank’s situation in advance greatly helps inanswering the questions.

• Always turn negatives into positives. Knowing who doesn’t pair up with Frank isnot nearly as valuable as knowing who does or can (in this case, Sally or one of the“X” plumbers).

Page 78: 15-PrepTest 15 Expls - dl.keywin.orgdl.keywin.org/3/4/340700517960b9bee3575f6766ff5765.pdf · • Sometimes the LSAT writers throw you a real curve when a Reading Comp ... LSAT Test

LSAT PREP ______________________________________________________________ LSAT Test XV Explained: Section IV

76 © K A P L A N

The Questions:

20. (C)Here’s our deduction about Frank. The only inexperienced plumber that Frank can teamup with is Sally, choice (C). If you don’t remember how we deduced this, look back at ourwork under Key Deductions. (Note that Kathy, choice (A), is not an inexperiencedplumber.)

• This question shows why Kaplan stresses taking so much time up front. Just goingthrough that extra thought about Frank’s possibilities, turned this question intonearly a gimme.

21. (C)This is similar to an acceptability question. Rule 3 allows us to cross off (A) and (B). Rule 2says that each team must have at least one experienced plumber, so there goes (D) and (E).We’re left with (C).

• This certainly didn’t take the form of a normal “acceptability” question, but by usingthe familiar technique of checking rules against the choices, it can be answeredpretty quickly.

22. (B)Take it one step at a time: Tim is assigned to a team, and Sally is assigned to another. Sally’spartner is a plumber who could have been Tim’s partner. Tim can only partner with Geneand Kathy. So of G and K, one of them will go with Tim and the other will go with Sally.Since Sally isn’t teamed with Frank, then Frank can only be teamed with anotherexperienced plumber (besides Gene and Kathy who are already taken). Mark is no goodthanks to Rule 3, which leaves only Jill, choice (B).

• Take questions with multiple hypotheticals apart; handle one thing at a time.

• Don’t be intimidated by a complex-looking stem; often, the more information theygive you, the better off you are.

Page 79: 15-PrepTest 15 Expls - dl.keywin.orgdl.keywin.org/3/4/340700517960b9bee3575f6766ff5765.pdf · • Sometimes the LSAT writers throw you a real curve when a Reading Comp ... LSAT Test

LSAT PREP ______________________________________________________________ LSAT Test XV Explained: Section IV

© K A P L A N 77

23. (A)If Gene is out, we know that 1) all of the other plumbers are used, and 2) Tim’s partner isKathy (see Rule 4). Since exactly four experienced plumbers are used, we must split themall up. Frank must therefore be teamed with an inexperienced one, which means Sally.Rule 5 says that Jill and Roberta can’t be teamed up, so Jill must be teamed with the onlyinexperienced plumber left, Vernon choice (A).

24. (A)All of the inexperienced plumbers are used, so one of our experienced plumbers is the oneto be left out. R, T, and V aren’t with Gene, so either Gene is teamed with Sally or Gene isthe one left out. If Gene is teamed with Sally, then Frank’s only possible inexperiencedpartner is taken, which means that he’s the one left out. Sally must therefore be teamed witheither Frank or Gene, choice (A).

• Notice how helpful our Frank deduction was to this game: It directly helped usanswer Q. 20 and indirectly played a role in answering Qs. 22, 23 and 24. (In Q. 21 youcould get by without it by simply referencing Rule 3.) This shows that the value of aKey Deduction extends far beyond simply helping to answer the one question thatdirectly tests for it.

Page 80: 15-PrepTest 15 Expls - dl.keywin.orgdl.keywin.org/3/4/340700517960b9bee3575f6766ff5765.pdf · • Sometimes the LSAT writers throw you a real curve when a Reading Comp ... LSAT Test

I.N. LL3113Rev.A Printed in the USA